Site Loader

Содержание

Эдс формула: схема, расчет, рисунок, как вычислить?

  

В разгар учебного года многим ученым деятелям требуется эдс формула для разных расчетов. Эксперименты, связанные с гальваническим элементом, так же нуждаются в информации об электродвижущей силе. Но для начинающих не так-то просто понять, что же это такое.

Формула нахождения эдс

Первым делом разберемся с определением. Что означает эта аббревиатура?

ЭДС или электродвижущая сила – это параметр характеризующий работу любых сил не электрической природы, работающих в цепях где сила тока как постоянного, так и переменного одинакова по всей длине. В сцепленном токопроводящем контуре ЭДС приравнивается работе данных сил по перемещению единого плюсового (положительного) заряда вдоль всего контура.

Ниже на рисунке представлена эдс формула.

Аст – означает работу сторонних сил в джоулях.

q это переносимый заряд в кулонах.

Сторонние силы – это силы которые выполняют разделение зарядов в источнике и в итоге образуют на его полюсах разность потенциалов.

Для этой силы единицей измерения является вольт. Обозначается в формулах она буквой «E».

Только в момент отсутствия тока в батареи, электродвижущая си-а будет равна напряжению на полюсах.

ЭДС индукции:

ЭДС индукции в контуре, имеющем N витков:

При движении:

 

Электродвижущая сила индукции в контуре, крутящемся в магнитном поле со скоростью w:

Таблица значений

Простое объяснение электродвижущей силы

Предположим, что в нашей деревне имеется водонапорная башня. Она полностью наполнена водой. Будем думать, что это обычная батарейка. Башня — это батарейка!

Вся вода будет оказывать сильное давление на дно нашей башенки. Но сильным оно будет только тогда, когда это строение полностью наполнено h3O.

В итоге чем меньше воды, тем слабее будет давление и напор струи будет меньше. Открыв кран, заметим, что каждую минуту дальность струи будет сокращаться.

В результате этого:

  1. Напряжение – это сила с которой вода давит на дно. То есть давление.
  2. Нулевое напряжение — это дно башни.

С батареей все аналогично.

Первым делом подключаем источник с энергией в цепь. И соответственно замыкаем ее. Например, вставляем батарею в фонарик и включаем его. Изначально заметим, что устройство горит ярко. Через некоторое время его яркость заметно понизится. То есть электродвижущая сила уменьшилась (вытекла если сравнивать с водой в башне).

Если брать в пример водонапорную башню, то ЭДС это насос качающие воду в башню постоянно. И она там никогда не заканчивается.

Эдс гальванического элемента формула

Электродвижущую силу батарейки можно вычислить двумя способами:

  • Выполнить расчет с применением уравнения Нернста. Нужно будет рассчитать электродные потенциалы каждого электрода, входящего в ГЭ. Затем вычислить ЭДС по формуле .
  • Посчитать ЭДС формуле Нернста для суммарной ток образующей реакции, протекающей при работе ГЭ.

Таким образом вооружившись данными формулами рассчитать электродвижущую силу батарейки будет проще.

Где используются разные виды ЭДС?
  1. Пьезоэлектрическая применяется при растяжении или сжатии материала. С помощью нее изготавливают кварцевые генераторы энергии и разные датчики.
  2. Химическая используется в гальванических элементах и аккумуляторах.
  3. Индукционная появляется в момент пересечения проводником магнитного поля. Ее свойства применяют в трансформаторах, электрических двигателях, генераторах.
  4. Термоэлектрическая образуется в момент нагрева контактов разнотипных металлов. Свое применение она нашла в холодильных установках и термопарах.
  5. Фото электрическая используется для продуцирования фотоэлементов.

 

 

Загрузка…

Эдс формула и ее расчеты. Напряжение при заряде и разряде Эдс батареи

Электродвижущая сила

Электродвижущей силой (ЭДС) аккумулятора Е называют разность его электродных потенциалов, измеренную при разомкнутой внешней цепи.

ЭДС батареи, состоящей из n последовательно соединенных аккумуляторов.

Следует различать равновесную ЭДС аккумулятора и неравновесную ЭДС аккумулятора в течение времени от размыкания цепи до установления равновесного состояния (период протекания переходного процесса). ЭДС измеряют высокоомным вольтметром (внутреннее сопротивление не менее 300 Ом/В). Для этого вольтметр присоединяют к выводам аккумулятора или батареи. При этом через аккумулятор (батарею) не должен протекать зарядный или разрядный ток.

Равновесная ЭДС свинцового аккумулятора, как и любого химического источника тока, зависит от химических и физических свойств веществ, принимающих участие в токообразующем процессе, и совершенно не зависит от размеров и формы электродов, а также от количества активных масс и электролита. Вместе с тем в свинцовом аккумуляторе электролит принимает непосредственное участие в токообразующем процессе на аккумуляторных электродах и изменяет свою плотность в зависимости от степени заряженности аккумуляторов. Поэтому равновесная ЭДС, которая в свою очередь является функцией плотности

Изменение ЭДС аккумулятора от температуры весьма мало и при эксплуатации им можно пренебречь.

Напряжение при заряде и разряде

Разность потенциалов на полюсных выводах аккумулятора (батареи) в процессе заряда или разряда при наличии тока во внешней цепи принято называть напряжением аккумулятора (батареи). Наличие внутреннего сопротивления аккумулятора приводит к тому, что его напряжение при разряде всегда меньше ЭДС, а при заряде — всегда больше ЭДС.

При заряде аккумулятора напряжение на его выводах должно быть больше его ЭДС на сумму внутренних потерь. В начале заряда происходит скачок напряжения на величину омических потерь внутри аккумулятора, а затем резкое повышение напряжения за счет потенциала поляризации, вызванное в основном быстрым увеличением плотности электролита в порах активной массы. Далее происходит медленный рост напряжения, обусловленный главным образом ростом ЭДС аккумулятора вследствие увеличения плотности электролита.

После того, как основное количество сульфата свинца преобразуется в РЬО2 и РЬ, затраты энергии все в большей мере вызывают разложение воды (электролиз) Избыточное количество ионов водорода и кислорода, появляющееся в электролите, еще больше увеличивает разность потенциалов разноименных электродов. Это приводит к быстрому росту зарядного напряжения, вызывающему ускорение процесса разложения воды. Образующиеся при этом ионы водорода и кислорода не вступают во взаимодействие с активными материалами. Они рекомбинируют в нейтральные молекулы и выделяются из электролита в виде пузырьков газа (на положительном электроде выделяется кислород, на отрицательном — водород), вызывая «кипение» электролита.

Если продолжить процесс заряда, можно увидеть, что рост плотности электролита и зарядного напряжения практически прекращается, так как уже почти весь сульфат свинца прореагировал, и вся подводимая к аккумулятору энергия теперь расходуется только на протекание побочного процесса — электролитическое разложение воды. Этим объясняется и постоянство зарядного напряжения, которое служит одним из признаков окончания зарядного процесса.

После прекращения заряда, то есть отключения внешнего источника, напряжение на выводах аккумулятора резко снижается до значения его неравновесной ЭДС, или на величину омических внутренних потерь. Затем происходит постепенное снижение ЭДС (вследствие уменьшения плотности электролита в порах активной массы), которое продолжается до полного выравнивания концентрации электролита в объеме аккумулятора и порах активной массы, что соответствует установлению равновесной ЭДС.

При разряде аккумулятора напряжение на его выводах меньше ЭДС на величину внутреннего падения напряжения.

В начале разряда напряжение аккумулятора резко падает на величину омических потерь и поляризации, обусловленной снижением концентрации электролита в порах активной массы, то есть концентрационной поляризации. Далее при установившемся (стационарном) процессе разряда происходит снижение плотности электролита в объеме аккумулятора, обусловливающее постепенное снижение разрядного напряжения. Одновременно происходит изменение соотношения содержания сульфата свинца в активной массе, что также вызывает повышение омических потерь. При этом частицы сульфата свинца (имеющего примерно втрое больший объем в сравнении с частицами свинца и его двуокиси, из которых они образовались) закрывают поры активной массы, чем препятствуют прохождению электролита в глубину электродов. Это вызывает усиление концентрационной поляризации, приводящее к более быстрому снижению разрядного напряжения.

При прекращении разряда напряжение на выводах аккумулятора быстро повышается на величину омических потерь, достигая значения неравновесной ЭДС. Дальнейшее изменение ЭДС вследствие выравнивания концентрации электролита в порах активных масс и в объеме аккумулятора приводит к постепенному установлению значения равновесной ЭДС.

Напряжение аккумулятора при его разряде определяется в основном температурой электролита и силой разрядного тока. Как сказано выше, сопротивление свинцового аккумулятора (батареи) незначительно и в заряженном состоянии составляет всего несколько миллиОм. Однако при токах стартерного разряда, сила которых в 4-7 раз превышает значение номинальной емкости, внутреннее падение напряжения оказывает существенное влияние на разрядное напряжение. Увеличение омических потерь с понижением температуры связано с ростом сопротивления электролита. Кроме того, резко возрастает вязкость электролита, что затрудняет процесс диффузии его в поры активной массы и повышает концентрационную поляризацию (то есть увеличивает потери напряжения внутри аккумулятора за счет снижения концентрации электролита в порах электродов). При токе более 60 А зависимость напряжения разряда от силы тока является практически линейной при всех температурах.

Среднее значение напряжения аккумулятора при заряде и разряде определяют как среднее арифметическое значений напряжения, измеренных через равные промежутки времени

Аккумулятор (элемент) — состоит из положительных и отрицательных электродов (свинцовых пластин) и сепараторов разделяющих эти пластины, установленных в корпус и погруженных в электролит (раствор серной кислоты). Накопление энергии в аккумуляторе происходит при протекании химической реакции окисления — восстановления электродов.

Аккумуляторная батарея состоит из 2 и более последовательно или (и) параллельно соединенных между собой секций (аккумуляторов, элементов) для обеспечения нужного напряжения и тока. Она способна накапливать, хранить и отдавать электроэнергию, обеспечивать запуск двигателя, а также питать электроприборы при неработающем двигателе.

Свинцово-кислотная аккумуляторная батарея — аккумуляторная батарея, в которой электроды изготовлены главным образом из свинца, а электролит представляет собой раствор серной кислоты.

Активная масса

— это составляющая часть электродов, которая претерпевает химические изменения при прохождении электрического тока во время заряда-разряда.

Электрод — проводящий материал, способный при реакции с электролитом производить электрический ток.

Положительный электрод (анод) — электрод (пластина) активная масса которого у заряженной батареи состоит из двуокиси свинца (PbO2).

Отрицательный электрод (катод) — электрод, активная масса которого у заряженной батареи состоит из губчатого свинца.

Решетка электрода служит для удерживания активной массы, а также для подвода и отвода тока к ней.

Сепаратор — материал, используемый для изоляции электродов друг от друга.

Полюсные выводы служат для подвода зарядного тока и для его отдачи под общим напряжением батареи.

Свинец — (Рb) — химический элемент четвертой группы периодической системы Д. И. Менделеева, порядковый номер 82, атомный вес 207,21, валентность 2 и 4. Свинец — синевато — серый металл, удельный вес его, в твердом виде составляющий 11,3 г/см 3 , уменьшается при расплавлении в зависимости от температуры. Самый пластичный среди металлов, он хорошо прокатывается до тончайшего листа и легко куётся. Свинец легко подвергается механической обработке, относится к числу легкоплавких металлов.

Окси́д свинца́(IV) (диоксид свинца́) PbO 2 представляет собой тёмно-коричневый тяжёлый порошок, имеющий тонкий характерный запах озона .

Сурьма представляет собой металл серебристо-белого цвета с сильным блеском, кристаллического строения. В противоположность свинцу — это твердый металл, но очень хрупкий и легко дробящийся на куски. Сурьма значительно легче свинца, ее удельный вес 6,7 г/см 3 . Вода и слабые кислоты на сурьму не действуют. Она медленно растворяется в крепкой соляной и серной кислотах.

Пробки ячеек закрывают отверстия ячеек в крышке батареи.

Пробка центральной вентиляции служит для перекрытия газоотводного отверстия в крышке батареи.

Моноблок — это полипропиленовый корпус батареи, разделенный перегородками на отдельные ячейки.

Дистиллированная вода доливается в батарею для возмещения ее потерь в результате разложения воды или испарения. Для доливки аккумуляторных батарей следует использовать только дистиллированную воду!

Электролит представляет собой раствор серной кислоты в дистиллированной воде, который заполняет свободные объемы ячеек и проникает в поры активной массы электродов и сепараторов.

Он способен проводить электрический ток между погруженными в него электродами. (Для средней полосы России плотностью 1.27-1.28 г/см3 при t=+20°С).

Малоподвижный электролит: Чтобы снизить опасность от вылившегося из батареи электролита, применяют средства, снижающие его текучесть. К электролиту могут быть добавлены вещества, которые превращают его в гель. Другим способом снижения подвижности электролита является применение стекломатов в качестве сепараторов.

Открытый аккумулятор — аккумулятор, имеющий пробку с отверстием, через которое доливается дистиллированная вода, и удаляются газообразные продукты. Отверстие может быть снабжено системой вентиляции.
Закрытый аккумулятор — аккумулятор, который закрыт в обычных условиях, но имеет устройство, позволяющее выделяться газу, когда внутреннее давление превышает установленное значение. Обычно дополнительная заливка электролита в такой аккумулятор невозможна.
Сухозаряженная батарея — аккумуляторная батарея, хранящаяся без электролита, пластины (электроды) которой находятся в сухом заряженном состоянии.

Трубчатая (панцирная) пластина — положительная пластина (электрод), которая состоит из комплекта пористых трубок, заполненных активной массой.

Предохранительный клапан — деталь вентиляционной пробки, которая позволяет выходить газу в случае избыточного внутреннего давления, но не допускает поступления воздуха в аккумулятор.

Ампер-час (А·ч) — это мера электрической энергии, равная произведению силы тока в амперах на время в часах (емкости).

Напряжение аккумулятора — разность потенциалов между выводами аккумулятора при разряде.
Емкость аккумуляторной батареи — количество электрической энергии, отдаваемой полностью заряженным аккумулятором при его разряде до достижения конечного напряжения.

Внутреннее сопротивление — сопротивление току через элемент, измеренное в Омах. Оно складывается из сопротивления электролита, сепараторов и пластин. Главной составляющей является сопротивление электролита, которое изменяется с изменением температуры и концентрации серной кислоты.

Плотность электролита — э то характеристика физического тела, равная отношению его массы к занимаемому объему. Она измеряется, например, в кг/л или в г/см3.

Срок службы батареи — период полезной работы батареи в заданных условиях.
Газовыделение — газообразование в процессе электролиза электролита.

Саморазряд — самопроизвольная потеря емкости аккумулятором в покое. Скорость саморазряда зависит от материала пластин, химических примесей в электролите, его плотности, от чистоты батареи и продолжительности ее эксплуатации.

ЭДС батареи (электродвижущая сила) — это напряжение на полюсных выводах полностью заряженной аккумуляторной батареи при разомкнутой цепи, т. е. при полном отсутствии токов заряда или разряда.

Цикл — одна последовательность заряда и разряда элемента.

Образование газов на электродах свинцового аккумулятора. Особенно обильно выделяется в конечной фазе заряда свинцового аккумулятора.

Гелевые аккумуляторы — это герметизированные свинцово-кислотные аккумуляторы (не герметичные, т.к. небольшое выделение газов при открытии клапанов все-таки происходит), закрытые, полностью необслуживаемые (недоливаемые) с гелеобразным кислотным электролитом (технологии Dryfit и Gelled Electrolite-Gel).

Технология AGM (Absorbed Glass Mat) — впитывающие прокладки из стекловолокна.

Отдача по энергии — отношение количества энергии, отдаваемой при разряде аккумулятора, к количеству энергии, необходимой для заряда до первоначального состояния при определенных условиях. Отдача по энергии для кислотных аккумуляторов при обычных условиях эксплуатации равна 65%, а для щелочных 55 — 60%.
Энергия удельная — энергия, отдаваемая аккумулятором при разряде в расчете на единицу его объема V или массы m, т. е. W= W/V или W= W/m. Удельная энергия кислотных аккумуляторов равна 7-25, никель-кадмиевых 11-27, никель-железных 20-36, серебряно-цинковых 120-130 Вт*ч/кг.

Короткое замыкание в аккумуляторах происходит при электрическом соединении пластин разной полярности.

ЭДС аккумулятора (Электродвижущая сила) это разность электродных потенциалов при отсутствии внешней цепи. Электродный потенциал складывается из равновесного электродного потенциала. Он характеризует состояние электрода в состоянии покоя, то есть отсутствии электрохимических процессов, и потенциала поляризации, определяющемуся как разность потенциалов электрода при зарядке (разрядке) и при отсутствии цепи.

Процесс диффузии.

Благодаря процессу диффузии, выравниванию плотности электролита в полости корпуса аккумулятора и в порах активной массы пластин, электродная поляризация может сохраняться в аккумуляторе при отключении внешней цепи.

Скорость прохождения диффузии напрямую зависит от температуры электролита, чем выше температура, тем быстрее проходит процесс и может сильно отличаться по времени, от двух часов до суток. Наличие двух составляющих электродного потенциала при переходных режимах привело к разделению на равновесную и не равновесную ЭДС аккумулятора .
На равновесную ЭДС аккумулятора влияет содержание и концентрация ионов активных веществ в электролите, а так же химические и физические свойства активных веществ. Главную роль в величине ЭДС играет плотность электролита и практически не влияет на неё температура. Зависимость ЭДС от плотности можно выразить формулой:

Где Е – ЭДС аккумулятора (В)

Р – плотность электролита приведённая к температуре 25 гр. С (г/см3) Эта формула истинна при рабочей плотности электролита в пределах 1,05 – 1,30 г/см3. ЭДС не может характеризовать степень разрежённости аккумулятора напрямую. Но если замерить его на выводах и сравнить с расчётным по плотности, то можно, с долей вероятности, судить о состоянии пластин и ёмкости.
В состоянии покоя плотность электролита в порах электродов и полости моноблока одинаковы и равны ЭДС покоя. При подключении потребителей или источника заряда, изменяется поляризация пластин и концентрация электролита в порах электродов. Это приводит к изменению ЭДС. При заряде значение ЭДС увеличивается, а при разряде уменьшается. Это связано с изменением плотности электролита, который участвует в электрохимических процессах.

Страница 2 из 26

1.3. Основные электрические характеристики аккумуляторных батарей

Электродвижущая сила и напряжение . Электродвижущей силой (ЭДС) называется разность потенциалов положительного и отрицательного электродов аккумулятора при разомкнутой внешней цепи.
Величина ЭДС зависит, главным образом, от электродных потенциалов, т. е. от физических и химических свойств веществ, из которых изготовлены пластины и электролит, но не зависит от размеров пластин аккумулятора.
ЭДС кислотного аккумулятора зависит также от плотности электролита. Теоретически и практически установлено, что ЭДС аккумулятора с достаточной для практики точностью можно определить по формуле
Е=0,85 + g,
где g– плотность электролита при 15°С, г/см 3 .
Для кислотных стартерных аккумуляторов, в которых плотность электролита колеблется в пределах от 1,12 до 1,29 г/см 3 , ЭДС изменяется соответственно от 1,97 до 2,14 В.
Измерить ЭДС с абсолютной точностью почти невозможно. Однако для практических целей ЭДС приблизительно и достаточно точно можно измерить вольтметром, имеющим высокое внутреннее сопротивление (не менее 1000 Ом на 1 В). При этом через вольтметр будет проходить ток незначительной величины.
Напряжением аккумулятора называется разность потенциалов положительных и отрицательных пластин при замкнутой внешней цепи, в которую включен какой-либо потребитель тока, т. е. при прохождении тока через аккумулятор. При этом показания вольтметра при измерении напряжения всегда будут меньше, чем при замере ЭДС, и эта разность будет тем больше, чем больший ток проходит через аккумулятор.
ЭДС и напряжение зависят от ряда факторов. ЭДС изменяется от плотности и температуры электролита. Напряжение в свою очередь зависит от ЭДС, величины разрядного тока (нагрузки) и внутреннего сопротивления аккумулятора.
Зависимость ЭДС аккумулятора от плотности электролита (концентрации раствора Н2SО4) приведена ниже:

Плотность электролита при 25°С,
г/см 3 ……………………………… 1,05 1,10 1,15 1,20 1,25 1,28 1,30
Н2SО4, %……………………….. 7,44 14,72 21,68 27,68 33,8 37,4 39,7
ЭДС аккумулятора, в………. 1,906 1,960 2,005 2,048 2,095 2,125 2,144
Из этой зависимости видно, что с увеличением концентрации серной кислоты ЭДС также увеличивается. Отсюда, однако, не следует, что для получения большей ЭДС можно чрезмерно увеличивать плотность электролита. Установлено, что стартерные аккумуляторные батареи достаточно хорошо работают тогда, когда плотность электролита в них составляет 1,27 – 1,29 г/см 3 .Кроме того, электролит плотностью 1,29 г/см 3 имеет самую низкую точку замерзания.
При изменении температуры электролита ЭДС аккумулятора также меняется. Так, с изменением температуры электролита от +20°С до -40°С ЭДС аккумулятора снижается с 2,12 до 2,096 в. В значительно большей степени с изменением температуры электролита меняется напряжение, так как оно зависит не только от ЭДС, но и от внутреннего сопротивления аккумулятора, которое с понижением температуры значительно возрастает.
Между ЭДС, напряжением, внутренним сопротивлением и величиной разрядного тока существует следующая зависимость:
U=Е-Ir,
где U – напряжение;
Е – э. д. с. аккумулятора;
I – величина разрядного тока;
r – внутреннее сопротивление аккумулятора.
Из этой формулы видно, что при постоянном значении ЭДС, измеряемой при разомкнутой цепи, напряжение аккумулятора падает по мере увеличения отдаваемого в процессе разряда тока.
Внутреннее сопротивление. Внутреннее сопротивление аккумулятора сравнительно мало, но в тех случаях, когда аккумуляторная батарея разряжается силой тока большой величины, например, при пуске двигателя стартером, внутреннее сопротивление каждого аккумулятора имеет очень существенное значение.
Внутреннее сопротивление складывается из сопротивления электролита, сепараторов и пластин. Главной составляющей является сопротивление электролита, которое изменяется с изменением температуры и концентрации серной кислоты.
Зависимость удельного сопротивления электролита плотностью 1,30 г/см 3 от температуры показана ниже:

Температура, °С Удельное сопротивление электролита Ом·см
+ 40 0,89
+ 25 1,28
+ 18 1,46
0 1,92
– 18 2,39
Как видно из приведенных данных, с понижением температуры электролита от +40°С до -18°С удельное сопротивление возрастает в 2,7 раза. Наименьшее значение удельного сопротивления имеет электролит плотностью 1,223 г/см 3 при 15°С (30%-ный раствор Н2SО4 по весу).
Вторым составляющим сопротивления в аккумуляторе является сопротивление сепараторов. Оно зависит в основном от их пористости. Сепараторы изготавливают из электроизолирующего материала, поры которого заполнены электролитом, что и обусловливает электропроводимость сепаратора.
В связи с этим можно было бы предположить, что с изменением температуры сопротивление сепаратора будет изменяться в той же пропорции, что и сопротивление электролита, но это не совсем так. Некоторые виды сепараторов, например, сепараторы из микропористого эбонита (мипора) не чувствительны к изменению температуры.
Третьим фактором, входящим в общую сумму внутреннего сопротивления элемента, служит активная масса и решетки положительных и отрицательных пластин.
Сопротивление губчатого свинца отрицательной пластины незначительно отличается от сопротивления материала решетки, в то время как сопротивление перекиси свинца положительной пластины превышает сопротивление решетки в 10000 раз. В отличие от сопротивления электролита сопротивление решетки уменьшается с понижением температуры. Но ввиду того, что сопротивление электролита во много раз больше сопротивления пластин, то уменьшение их сопротивления с понижением температуры весьма незначительно компенсирует общее снижение сопротивления электролита.
На сопротивление пластин влияет степень заряженноcти аккумуляторной батареи. В процессе разряда сопротивление пластин возрастает, так как сернокислый свинец, образующийся на положительных и отрицательных пластинах, почти не проводит электрический ток.
По сравнению с другими типами аккумуляторов кислотные аккумуляторы имеют сравнительно малое внутреннее сопротивление, что и определяет их широкое применение в качестве стартерных батарей на автомобильном транспорте.
Емкость. Емкостью аккумулятора называется количество электричества, которое может отдать полностью заряженный аккумулятор при заданном режиме разряда, температуре и конечном напряжении. Емкость измеряют в ампер-часах и определяют по формуле
C=Iptp,
где С – емкость, а·ч;
Ip – сила разрядного тока, а;
tp – время разряда, ч.
Величина емкости аккумуляторной батареи в основном определяется следующими факторами: режимом разряда (величиной разрядного тока), концентрацией электролита и температурой. Аккумуляторы при форсированных режимах разряда отдают емкость меньше, чем при разряде более длительными режимами (небольшой величиной тока).
Снижение емкости при форсированных режимах разряда происходит по следующим причинам.
В процессе разряда превращение активной массы пластин сернокислый свинец происходит не только на поверхности пластин, но и внутри них. Если разряд осуществляют током небольшой силы и медленно, то электролит успевает проникать в глубокие слои активной массы, а вода, образующаяся в результате реакции в порах, успевает смешаться с основной массой электролита. При форсированных режимах разряда концентрация серной кислоты в электролите внутри пластин значительно снижается, свежий электролит не успевает проникнуть в глубь активной массы, реакция идет в основном на поверхности пластин, так как поры закупориваются и внутрилежащие слои активной массы почти не принимают участия в реакции. При этом в результате значительного увеличения внутреннего сопротивления аккумулятора напряжение на его зажимах резко падает.
Однако после того как аккумулятор будет разряжен при форсированном режиме, после небольшого перерыва его снова можно разряжать. Это служит наглядным подтверждением того, что снижение емкости в аккумуляторе при разряде большой величиной силы тока происходит в результате неполного использования активной массы пластин.
Кроме величины разрядного тока, на емкость аккумулятора значительно влияет концентрация электролита, которая определяет потенциал пластин, электрическое сопротивление электролита и его вязкость, влияющую в свою очередь на способность проникания электролита в глубокие слои активной массы пластин.
В процессе разряда плотность электролита уменьшается и в конце разряда к активной массе пластин поступает недостаточное количество кислоты, в результате чего напряжение аккумулятора падает и дальнейший его разряд становится невозможным. Чем больше разница между концентрациями электролита, находящегося вне пластин, и электролита, находящегося в порах активной массы, тем интенсивнее происходит процесс проникновения кислоты в поры пластин. В этом отношении применение электролита с большей плотностью, казалось бы, должно увеличить емкость. Но в действительности чрезмерно большая плотность не ведет к увеличению емкости, так как увеличение плотности электролита неизбежно приводит к повышению вязкости электролита, в результате чего процесс проникновения электролита в глубину активной массы пластин ухудшается, и напряжение на зажимах аккумулятора падает.
Установлено, что наибольшую емкость имеет аккумуляторная батарея с плотностью электролита 1,27 – 1,29 г/см 3 .
Емкость аккумуляторной батареи зависит также от температуры. С понижением температуры емкость снижается, а с повышением увеличивается. Это объясняется тем, что с понижением температуры увеличивается вязкость электролита, в результате чего он поступает к пластинам в недостаточном количестве.
Значения вязкости электролита плотностью 1,223 г/см 3 в зависимости от температуры приведены ниже:
Температура, °С………… +30 +25 +20 +10 0 – 10 – 20 – 30
Абсолютная вязкость,
пз(пуаз)………………….. 1,596 1,784 2,006 2,600 3,520 4,950 7,490 12,200
Емкость положительных и отрицательных пластин с изменением температур изменяется не в одинаковой степени. Если при обычной температуре емкость элемента лимитируется положительными пластинами, то при низких температурах – отрицательными, так как при понижении температуры емкость отрицательной пластины уменьшается в значительно большей степени, чем положительной.
В последнее время емкость аккумуляторных батарей при низких температурах удалось значительно повысить за счет применения более тонких синтетических сепараторов с высокой пористостью (до 80%) и присадок, так называемых расширителей, к активной массе отрицательных пластин, которые придают ей большую пористость.
Помимо режима разряда, концентрации электролита и температуры емкость аккумуляторной батареи зависит от срока ее службы, от срока хранения, в течение которого батарея бездействовала, от наличия вредных примесей и т. д. Емкость новой аккумуляторной батареи, поступающей в эксплуатацию, первое время (в течение гарантийного срока службы) повышается, так как происходит формирование пластин, после чего на протяжении определенного периода остается постоянной и затем начинает постепенно падать. Потеря емкости аккумуляторной батареей в конце срока службы объясняется уменьшением пористости отрицательных пластин и выпадением активной массы положительных пластин.
Если заряженная батарея продолжительное время бездействовала, то при ее разряде отданная емкость будет значительно меньше. Это объясняется естественным явлением саморазряда при бездействии батареи.

Давайте рассмотрим основные параметры аккумулятора, которые понадобяться нам при его эксплуатации.

1. Электродвижущая сила (ЭДС) аккумуляторной батареи — напряжение между выводами аккумуляторной батареи при разомкнутой внешней цепи (и, конечно-же, при отсутствии каких-либо утечек). В «полевых» условиях (в гараже) ЭДС можно измерить любым тестером, перед этим сняв одну из клемм («+» или «-») с аккумулятора.

ЭДС аккумулятора зависит от плотности и от температуры электролита и совершенно не зависит от размеров и формы электродов, а также от количества электролита и активных масс. Изменение ЭДС аккумулятора от температуры весьма мало и при эксплуатации им можно пренебречь. С повышением плотности электролита ЭДС повышается. При температуре плюс 18°С и плотности d = 1,28 г/см 3 аккумулятор (имеется в виду одна банка) обладает ЭДС рав­ной 2,12 В (АКБ — 6 х 2,12 В = 12,72 В). Зависимость ЭДС от плотности электролита при изме­нении плотности в пределах 1,05 ÷ 1,3 г/см 3 вы­ражается эмпирической формулой

Е=0,84+d , где

Е — ЭДС аккумулятора, В;

d — плотность электролита при температуре плюс 18°С, г/см 3 .

По ЭДС нельзя точно судить о степени разряженности ак­кумулятора. ЭДС разряженного аккумулятора с большей плот­ностью электролита будет выше, чем ЭДС заряженного акку­мулятора, но имеющего меньшую плотность электролита.

Путём измерения ЭДС можно только быстро обнаружить серьезную неисправность аккумуляторной батареи (замыкание пластин в одной или нескольких банках, обрыв соединительных проводников между банками и тому подобное).

2. Внутреннее сопротивление аккумулятора представляет собой сумму сопротивлений выводных зажимов, межэлементных соеди­нений, пластин, электролита, сепараторов и сопротивления, во­зникающего в местах соприкосновения электродов с электро­литом. Чем больше емкость аккумулятора (число пластин), тем меньше его внутреннее сопротивление. С понижением темпера­туры и по мере разряда аккумулятора его внутреннее сопротив­ление растет. Напряжение аккумулятора отличается от его ЭДС на величину падения напряжения на внутреннем сопротивлении ак­кумулятора.

При заряде U 3 = Е + I х R ВН ,

а при разряде U Р = Е — I х R ВН , где

I — ток, протекаю­щий через аккумулятор, A;

R ВН — внутреннее сопротивление акку­мулятора, Ом;

Е — ЭДС аккуму­лятора, В.

Изменение напряже­ния на аккумуляторной батарее при ее заряде и разряде показано на Рис. 1.

Рис.1. Изменение напряжения аккумуляторной батареи при её заряде и разряде.

1 — начало газовыделения, 2 — заряд, 3 — разряд.

Напряжение автомобильного генератора, от которого производится заряд батареи, составляет 14,0÷14,5 В . На автомобиле батарея, даже в лучшем случае, при полностью благоприятных условиях, остается недозаряженной на 10÷20% . Виной всему — работа автомобильного генератора.

Достаточное для зарядки напряжение генератор начинает выдавать при 2000 об/мин и более. Обороты холостого хода 800÷900 об/мин . Стиль езды в городе: разгон (длительность меньше минуты), торможение, остановка (светофор, пробка — длительность от 1 минуты до ** часов). Заряд идёт только во время разгона и движения на довольно высоких оборотах. В остальное время идёт интенсивный разряд АКБ (фары, прочие потребители электроэнергии, сигнализация — круглосуточно ).

Ситуация улучшается при движении за городом, но не критическим образом. Длительность поездок не так велика (полный заряд батареи — 12÷15 часов ).

В точке 1 — 14,5 В начинается газовыделение (электролиз воды на кислород и водород), увеличивается расход воды. Другой неприятный эффект при электролизе — увеличивается коррозия пластин, поэтому не следует допускать длительного превышения напряжения 14,5 В на клеммах АКБ.

Напряжение автомобильного генератора (14,0÷14,5 В ) выбрано из компромиссных условий — обеспечение более-менее нормальной зарядки батареи при уменьшении газообразования (снижается расход воды, понижается пожароопасность, уменьшается скорость разрушения пластин).

Из вышесказанного можно сделать вывод, что батарею нужно периодически, хотя бы раз в месяц, полностью дозаряжать внешним зарядным устройством для уменьшения сульфатации пластин и увеличения срока службы.

Напряжение аккумуляторной батареи при ее разряде стартерным током (I Р = 2÷ 5 С 20) зависит от силы раз­рядного тока и темпе­ратуры электролита. На Рис.2 показаны вольт-амперные харак­теристики аккумуля­торной батареи 6СТ-90 при различной темпе­ратуре электролита. Если разрядный ток будет постоянным (например, I Р = 3 С 20 , линия 1), то напряжение батареи при разряде будет тем меньше, чем ниже ее температура. Для сохранения по­стоянства напряжения при разряде (линия 2) необходимо с пониже­нием температуры ба­тареи снижать силу разрядного тока.

Рис.2. Вольт-амперные характеристики АКБ 6СТ-90 при различной температуре электролита.

3. Емкостью аккумулятора (С) называется количество электри­чества, которое аккумулятор отдает при разряде до наименьшего допустимого напряжения. Ёмкость аккумулятора выражается в Ампер-часах (А ч ). Чем больше сила разрядного тока, тем ниже напряжение, до которого может разряжаться аккумулятор, например при определении номинальной емкости аккумуляторной батареи разряд ведется током I = 0,05С 20 до напряжения 10,5 В , температура электролита должна быть в интервале +(18 ÷ 27)°С , а время разряда 20 ч . Считается, что конец срока службы батареи наступает, когда ее емкость составляет 40% от С 20 .

Емкость батареи в стартерных режимах определяется при температуре +25°С и разрядном токе ЗС 20 . В этом случае время разряда до напряжения 6 В (один вольт на аккумулятор) дол­жно быть не менее 3 мин .

При разряде батареи током ЗС 20 (температура электро­лита -18°С ) напряже­ние батареи через 30 с после начала разряда должно быть 8,4 В (9,0 В для необслужи­ваемых батарей), а после 150 с не ниже 6 В . Этот ток иногда называют током холодной прокрутки или пусковым током , он может отличаться от ЗС 20 Этот ток указывается на корпусе батареи рядом с ее емкостью.

Если разряд происходит при постоянной силе тока, то ем­кость аккумуляторной батареи определяется по формуле

С = I х t где,

I — ток разряда, A;

t — время разряда, ч.

Емкость аккумуляторной батареи зависит от ее конструкции, числа пластин, их толщины, материала сепаратора, пористости активного материала, конструкции решетки пластин и других факторов. В эксплуатации емкость батареи зависит от силы разрядного тока, температуры, режима разряда (прерывистый или непрерывный), степени заряженности и изношенности акку­муляторной батареи. При увеличении разрядного тока и степени разряженности, а также с понижением температуры емкость ак­кумуляторной батареи уменьшается. При низких температурах падение емкости аккумуляторной батареи с повышением разряд­ных токов происходит особенно интенсивно. При температуре −20°С остается около 50% от емкости батареи при температуре +20°С.

Наиболее полно состояние аккумуляторной батареи показывает как раз её ёмкость. Для определения реальной емкости достаточно полностью заряженную исправную батарею поставить на разряд током I = 0,05 С 20 (например, для батареи с ёмкостью 55 Ач, I = 0,05 х 55 = 2,75 А). Разряд следует продолжать до достижения величины напряжения на батарее 10,5 В . Время разряда должно составить не менее 20 часов .

В качестве нагрузки при определении ёмкости удобно использовать автомобильные лампы накаливания . Например, чтобы обеспечить разрядный ток 2,75 А , при котором потребляемая мощность составит Р = I x U = 2,75 А x 12,6 В = 34,65 Вт , достаточно соединить параллельно лампу на 21 Вт и лампу на 15 Вт . Рабочее напряжение ламп накаливания для нашего случая должно быть 12 В . Конечно, точность установки тока подобным образом — «плюс-минус лапоть», но для приблизительного определения состояния аккумуляторной батареи вполне достаточно, а так-же дёшево и доступно.

При проверке таким образом новых батарей, время разряда может оказаться меньше 20 часов. Это обусловлено тем, что номинальную ёмкость они набирают после 3÷ 5 полных циклов заряд-разряд.

Ёмкость АКБ можно оценить также с помощью нагрузочной вилки . Нагрузочная вилка состоит из двух контактных ножек, рукоятки, переключаемого нагрузочного сопротивления и вольтметра. Один из возможных вариантов показан на Рис.3.

Рис.3. Вариант нагрузочной вилки .

Для проверки современных батарей, у которых доступны только выходные клеммы, надо использовать 12-ти вольтовые нагрузочные вилки . Нагрузочное сопротивление выбирается таким, чтобы обеспечить нагрузку аккумулятора током I = ЗС 20 (например, при ёмкости батареи 55 Ач, нагрузочное сопротивление должно потреблять ток I = ЗС 20 = 3 х 55 = 165 А). Нагрузочная вилка подсоединяется параллельно выходным контактам полностью заряженной батареи, замечается время, в течение которого выходное напряжение снизится от 12,6 В до 6 В . Это время у новой, исправной и полностью заряженной батареи должно быть не менее трёх минут при температуре электролита +25°С .

4. Саморазряд аккумулятора. Саморазрядом называют снижение емкости аккумуляторов при разомкнутой внешней цепи, то есть при бездействии. Это явление вызвано окислительно-восстановительными процессами, самопроизвольно протекающими как на отрицательном, так и на положительном электродах.

Саморазряду особенно подвержен отрицательный электрод вследствие самопроизвольного растворения свинца (отрицательной активной массы) в растворе серной кислоты.

Саморазряд отрицательного электрода сопровождается выделением газообразного водорода. Скорость самопроизвольного растворения свинца существенно возрастает с повышением концентрации электролита. Повышение плотности электролита с 1,27 до 1,32 г/см 3 приводит к росту скорости саморазряда отрицательного электрода на 40 %.

Саморазряд может возникать также, когда аккумулятор снаружи загрязнен или залит электролитом, водой или другими жидкостями, которые создают возможность разряда через электропроводную пленку, находящуюся между полюсными выводами аккумулятора или его перемычками.

Саморазряд батарей в значительной мере зависит от температуры электролита . С понижением температуры саморазряд уменьшается. При температуре ниже 0°С у новых батарей он практически прекращается. Поэтому хранение батарей рекомендуется в заряженном состоянии при низких температурах (до −30°С). Всё это показано на Рис.4 .

Рис.4. Зависимость саморазряда АКБ от температуры.

В процессе эксплуатации саморазряд не остается постоянным и резко усиливается к концу срока службы.

Для снижения саморазряда необходимо использовать возможно более чистые материалы для производства аккумуляторов, использовать только чистую серную кислоту и дистиллированную воду для приготовления электролита, как при производстве, так и при эксплуатации.

Обычно степень саморазряда выражают в процентах потери емкости за установленный период времени. Саморазряд аккумуляторов считается нормальным, если он не превышает 1% в сутки, или 30% емкости батареи в месяц.

5. Срок хранения новых батарей. В настоящее время автомобильные батареи выпускаются заводом-изготовителем только в сухозаряженном состоянии. Срок хранения батарей без эксплуатации весьма ограничен и не превышает 2 лет (гарантийный срок хранения 1 год ).

6. Срок службы автомобильных свинцово-кислотных аккумуляторных батарей — не менее 4-х лет при соблюдении установленных заводом условий эксплуатации. Из моей практики шесть батарей прослужили по четыре года, а одна, самая стойкая, — целых восемь лет.


Значит эдс. Что такое электродвижущая сила эдс. Определение и физический смысл

В разгар учебного года многим ученым деятелям требуется эдс формула для разных расчетов. Эксперименты, связанные с , так же нуждаются в информации об электродвижущей силе. Но для начинающих не так-то просто понять, что же это такое.

Формула нахождения эдс

Первым делом разберемся с определением. Что означает эта аббревиатура?

ЭДС или электродвижущая сила – это параметр характеризующий работу любых сил не электрической природы, работающих в цепях где сила тока как постоянного, так и переменного одинакова по всей длине. В сцепленном токопроводящем контуре ЭДС приравнивается работе данных сил по перемещению единого плюсового (положительного) заряда вдоль всего контура.

Ниже на рисунке представлена эдс формула.

Аст – означает работу сторонних сил в джоулях.

q – это переносимый заряд в кулонах.

Сторонние силы – это силы которые выполняют разделение зарядов в источнике и в итоге образуют на его полюсах разность потенциалов.

Для этой силы единицей измерения является вольт . Обозначается в формулах она буквой « E».

Только в момент отсутствия тока в батареи, электродвижущая си-а будет равна напряжению на полюсах.

ЭДС индукции:

ЭДС индукции в контуре, имеющем N витков:

При движении:

Электродвижущая сила индукции в контуре, крутящемся в магнитном поле со скоростью w :

Таблица значений

Простое объяснение электродвижущей силы

Предположим, что в нашей деревне имеется водонапорная башня. Она полностью наполнена водой. Будем думать, что это обычная батарейка. Башня — это батарейка!

Вся вода будет оказывать сильное давление на дно нашей башенки. Но сильным оно будет только тогда, когда это строение полностью наполнено H 2 O.

В итоге чем меньше воды, тем слабее будет давление и напор струи будет меньше. Открыв кран, заметим, что каждую минуту дальность струи будет сокращаться.

В результате этого:

  1. Напряжение – это сила с которой вода давит на дно. То есть давление.
  2. Нулевое напряжение — это дно башни.

С батареей все аналогично.

Первым делом подключаем источник с энергией в цепь. И соответственно замыкаем ее. Например, вставляем батарею в фонарик и включаем его. Изначально заметим, что устройство горит ярко. Через некоторое время его яркость заметно понизится. То есть электродвижущая сила уменьшилась (вытекла если сравнивать с водой в башне).

Если брать в пример водонапорную башню, то ЭДС это насос качающие воду в башню постоянно. И она там никогда не заканчивается.

Эдс гальванического элемента – формула

Электродвижущую силу батарейки можно вычислить двумя способами:

  • Выполнить расчет с применением уравнения Нернста. Нужно будет рассчитать электродные потенциалы каждого электрода, входящего в ГЭ. Затем вычислить ЭДС по формуле.
  • Посчитать ЭДС формуле Нернста для суммарной ток образующей реакции, протекающей при работе ГЭ.

Таким образом вооружившись данными формулами рассчитать электродвижущую силу батарейки будет проще.

Где используются разные виды ЭДС?
  1. Пьезоэлектрическая применяется при растяжении или сжатии материала. С помощью нее изготавливают кварцевые генераторы энергии и разные датчики.
  2. Химическая используется в и аккумуляторах.
  3. Индукционная появляется в момент пересечения проводником магнитного поля. Ее свойства применяют в трансформаторах, электрических двигателях, генераторах.
  4. Термоэлектрическая образуется в момент нагрева контактов разнотипных металлов. Свое применение она нашла в холодильных установках и термопарах.
  5. Фото электрическая используется для продуцирования фотоэлементов.

Электродвижущая сила (ЭДС) — в устройстве, осуществляющем принудительное разделение положительных и отрицательных зарядов (генераторе), величина, численно равная разности потенциалов между зажимами генератора при отсутствии тока в его цепи, измеряется в Вольтах.

Источники электромагнитной энергии (генераторы) — устройства, преобразующие энергию любого неэлектрического вида в электрическую. Такими источниками, например, являются :

    генераторы на электростанциях (тепловых, ветровых, атомных, гидростанциях), преобразующие механическую энергию в электрическую;

    гальванические элементы (батареи) и аккумуляторы всех типов, преобразующие химическую энергию в электрическую и т. п.

ЭДС численно равна работе, которую совершают сторонние силы при перемещении единичного положительного заряда внутри источника или сам источник, проводя единичный положительный заряд по замкнутой цепи.

Электродвижущая сила ЭДС Е — скалярная величина, характеризующая способность стороннего поля и индуктированного электрического поля вызывать электрический ток. ЭДС Е численно равна работе (энергии) W в джоулях (Дж), затрачиваемой этим полем на перемещение единицы заряда (1 Кл) из одной точки поля в другую.

Единицей измерения ЭДС является вольт (В). Таким образом, ЭДС равна 1 В, если при перемещении заряда в 1 Кл по замкнутой цепи совершается работа в 1 Дж: [Е] = I Дж/1 Кл = 1 В.

Перемещение зарядов по участку сопровождается затратой энергии.

Величину, численно равную работе, которую совершает источник, проводя единичный положительный заряд по данному участку цепи, называют напряжением U. Так как цепь состоит из внешнего и внутреннего участков, разграничивают понятия напряжений на внешнем Uвш и внутреннем Uвт участках.

Из сказанного очевидно, что ЭДС источника равна сумме напряжений на внешнем U и внутреннем U участках цепи:

Е = Uвш + Uвт.

Эта формула выражает закон сохранения энергии для электрической цепи.

Измерить напряжения на различных участках цепи можно только при замкнутой цепи. ЭДС измеряют между зажимами источника при разомкнутой цепи.


Направление ЭДС — это направление принудительного движения положительных зарядов внутри генератора от минуса к плюсу под действием иной, чем электрическая, природы.

Внутреннее сопротивление генератора это сопротивление конструктивных элементов внутри него.

Идеальный источник ЭДС — генератор, которого равно нулю, а напряжение на его зажимах не зависит от нагрузки. Мощность идеального источника ЭДС бесконечна.

Условное изображение (электрическая схема) идеального генератора ЭДС величиной Е показано на рис. 1, а.

Реальный источник ЭДС, в отличие от идеального, содержит внутреннее сопротивление Ri и его напряжение зависит от нагрузки (рис. 1., б), а мощность источника конечна. Электрическая схема реального генератора ЭДС представляет собой последовательное соединение идеального генератора ЭДС Е и его внутреннего сопротивления Ri.


На практике для того чтобы приблизить режим работы реального генератора ЭДС к режиму работы идеального, внутреннее сопротивление реального генератора Ri стараются делать как можно меньше, а сопротивление нагрузки Rн необходимо подключать величиной не менее чем в 10 раз большей величины внутреннего сопротивления генератора, т.е. необходимо выполнять условие: Rн >> Ri

Для того чтобы выходное напряжение реального генератора ЭДС не зависело от нагрузки, его стабилизируют применением специальных электронных схем стабилизации напряжения.

Поскольку внутреннее сопротивление реального генератора ЭДС не может быть выполнено бесконечно малым, его минимизируют и выполняют стандартным для возможности согласованного подключения к нему потребителей энергии. В радиотехнике величины стандартного выходного сопротивления генераторов ЭДС составляют 50 Ом (промышленный стандарт) и 75 Ом (бытовой стандарт).

Например, все телевизионные приемники имеют входное сопротивление 75 Ом и подключены к антеннам коаксиальным кабелем именно такого волнового сопротивления.

Для приближения к идеальным генераторам ЭДС источники питающего напряжения, используемые во всей промышленной и бытовой радиоэлектронной аппаратуре, выполняют с применением специальных электронных схем стабилизации выходного напряжения, которые позволяют выдерживать практически неизменное выходное напряжение источника питания в заданном диапазоне токов, потребляемых от источника ЭДС (иногда его называют источником напряжения).

На электрических схемах источники ЭДС изображаются так: Е — источник постоянной ЭДС, е(t) — источник гармонической (переменной) ЭДС в форме функции времени.

Электродвижущая сила Е батареи последовательно соединенных одинаковых элементов равна электродвижущей силе одного элемента Е, умноженной на число элементов n батареи: Е = nЕ.

Электродвижущая сила, в народе ЭДС, также как и напряжение измеряется в вольтах, но носит совсем иной характер.

ЭДС с точки зрения гидравлики

Думаю, вам уже знакома водонапорная башня из прошлой статьи про

Допустим, что башня полностью заполнена водой. Снизу башни мы просверлили отверстие и врезали туда трубу, по которой вода бежит к вам домой.


Сосед захотел полить огурцы, вы решили помыть автомобиль, мать затеяла стирку и вуаля! Поток воды стал меньше и меньше, и вскоре совсем иссяк… Что случилось? Закончилась вода в башне…


Время, которое потребуется, чтобы опустошить башню, зависит от емкости самой башни, а также от того, сколько потребителей будут пользоваться водой.

Все то же самое можно сказать и про радиоэлемент конденсатор :

Допустим мы его зарядили от батарейки 1,5 вольта и он принял заряд. Нарисуем заряженный конденсатор вот так:

Но как только мы цепляем к нему нагрузку (пусть нагрузкой будет светодиод) с помощью замыкания ключа S, в первые доли секунд светодиод будет светиться ярко, а потом тихонько угасать… и пока полностью не потухнет. Время угасания светодиода будет зависеть от емкости конденсатора, а также от того, какую нагрузку мы цепляем к заряженному конденсатору.

Как я уже сказал, это равносильно простой наполненной башне и потребителям, которые пользуются водой.

Но почему тогда в наших башнях вода никогда не заканчивается? Да потому что работает насос подачи воды ! А откуда этот насос берет воду? Из скважины, которая пробурена для добычи подземных вод. Иногда ее еще называют артезианской.


Как только башня полностью наполнится водой, насос выключается. В наших водобашнях насос всегда поддерживает максимальный уровень воды.

Итак, давайте вспомним, что такое напряжение ? По аналогии с гидравликой – это уровень воды в водобашне. Полная башня – это максимальный уровень воды, значит максимальное напряжение. Нет в башне воды – напряжение ноль.

ЭДС электрического тока

Как вы помните из прошлых статей, молекулы воды – это “электроны”. Для возникновения электрического тока, электроны должны двигаться в одном направлении. Но чтобы они двигались в одном направлении, должно быть напряжение и какая-нибудь нагрузка. То есть вода в башне – это напряжение, а люди, которые тратят воду для своих нужд – это нагрузка, так как они создают поток воды из трубы, которая находится у подножия водобашни. А поток – это не что иное, как сила тока.

Также должно соблюдаться условие, что вода должна всегда быть на максимальной отметке, независимо от того, сколько людей тратит ее для своих нужд одновременно, иначе башня опустошится. Для водобашни этим спасительным средством является водонасос. А для электрического тока?

Для электрического тока должна быть какая-то сила, которая бы толкала электроны в одном направлении в течение продолжительного времени. То есть эта сила должна двигать электроны! Электродвижущая сила! Да, именно так! ЭЛЕКТРОДВИЖУЩАЯ СИЛА! Можно назвать ее сокращенно ЭДС – Э лектро Д вижущая С ила. Измеряется она в вольтах, как и напряжение, и обозначается в основном буквой E .

Значит, в наших батарейках тоже есть такой “насос”? Есть, и правильней было бы его назвать “насос подачи электронов”). Но, конечно, так никто не говорит. Говорят просто – ЭДС. Интересно, а где спрятан этот насос в батарейке? Это просто-напросто электрохимическая реакция, из-за которой держится “уровень воды” в батарейке, но потом все-таки этот насос изнашивается и напряжение в батарейке начинает проседать, потому как “насос” не успевает качать воду. В конце концов он полностью ломается и напряжение на батарейке стает практически ноль.

Реальный источник ЭДС

Источник электрической энергии – это источник ЭДС с внутренним сопротивлением R вн. Это могут быть какие-либо химические элементы питания, наподобие батареек и аккумуляторов


Их внутреннее строение с точки зрения ЭДС выглядит примерно вот так:


Где E – это ЭДС, а R вн – это внутреннее сопротивление батарейки

Итак, какие выводы можно сделать из этого?

Если к батарейке не цепляется никакая нагрузка, типа лампы накаливания и тд, то в результате сила тока в такой цепи будет равняться нулю. Упрощенная схема будет такой:


Но если мы все-таки присоединим к нашей батарейке лампочку накаливания, то у нас цепь станет замкнутой и в цепи будет течь ток:

Если начертить график зависимости силы в цепи тока от напряжения на батарейке, то он будет выглядеть вот так:


Какой напрашивается вывод? Для того, чтобы замерить ЭДС батарейки, нам достаточно просто взять хороший мультиметр с высоким входным сопротивлением и замерять напряжение на клеммах батарейки.

Идеальный источник ЭДС

Допустим, пусть наша батарейка обладает нулевым внутренним сопротивлением, тогда получается, что R вн =0.

Нетрудно догадаться, что в этом случае падение напряжение на нулевом сопротивлении также будет равняться нулю. В результате, наш график примет вот такой вид:


В результате мы получили просто источник ЭДС. Следовательно, источник ЭДС – это идеальный источник питания, у которого напряжение на клеммах не зависит от силы тока в цепи. То есть, какую нагрузку мы бы не цепляли на такой источник ЭДС, у нас он все равно будет выдавать положенное напряжение без просадки. Сам источник ЭДС обозначается вот так:

На практике идеального источника ЭДС не существует.

Типы ЭДС

электрохимическая (ЭДС батареек и аккумуляторов)

фотоэффекта (получение электрического тока от солнечной энергии)

индукции (генераторы, использующие принцип электромагнитной индукции)

Эффект Зеебека или термоЭДС (возникновение электрического тока в замкнутой цепи, состоящей из последовательно соединённых разнородных проводников , контакты между которыми находятся при различных температурах)

пьезоЭДС (получение ЭДС от )

Резюме

ЭДС – это сила НЕэлектрического происхождения, которая заставляет течь электрический ток в цепи.

Реальный источник ЭДС имеет внутри себя внутреннее сопротивление, у идеального источника ЭДС внутреннее сопротивление равняется нулю.

Идеальный источник ЭДС всегда имеет на своих клеммах постоянное значение напряжения не зависимо от нагрузки в цепи.

Содержание:

Когда родилось понятие «электрон», люди сразу связали его с определенной работой. Электрон – это по-гречески «янтарь». То, что грекам для того, чтобы найти этот бесполезный, в общем-то, магический камушек, надо было довольно далеко проехать на север — такие усилия тут, в общем-то, не в счет. А вот стоило проделать некоторую работу — руками по натиранию камушка о шерстяную сухую тряпочку — и он приобретал новые свойства. Это знали все. Натереть просто так, ради сугубо бескорыстного интереса, чтобы понаблюдать, как теперь к «электрону» начинает притягиваться мелкий мусор: пылинки, шерстинки, ниточки, перышки. В дальнейшем, когда появился целый класс явлений, объединенных потом в понятие «электричество», работа, которую надо обязательно затратить, не давала людям покоя. Раз нужно затратить, чтобы получился фокус с пылинками — значит, хорошо бы эту работу как-то сохранить, накопить, а потом и получить обратно.

Таким образом из все более усложнявшихся фокусов с разными материалами и философских рассуждений и научились эту магическую силу собирать в баночку. А потом сделать и так, чтобы она из баночки постепенно высвобождалась, вызывая действия, которые стало уже можно ощутить, а очень скоро и померить. И померили настолько остроумно, имея всего-то пару шелковых шариков или палочек и пружинные крутильные весы, что и теперь мы вполне серьезно пользуемся все теми же формулами для расчетов электрических цепей, которые уже пронизали теперь всю планету, бесконечно сложных, сравнительно с теми первыми приспособлениями.

А название этого могучего джинна, сидящего в баночке, так до сих пор и содержит восторг давних открывателей: «Электродвижущая сила». Но только сила эта — совсем не электрическая. А наоборот, посторонняя страшная сила, заставляющая электрические заряды двигаться «против воли», то есть преодолевая взаимное отталкивание, и собираться где-то с одной стороны. От этого получается разность потенциалов. Ее и можно использовать, пустив заряды другим путем. Где их «не сторожит» эта страшная ЭДС. И заставить, тем самым, выполнить некоторую работу.

Принцип работы

ЭДС — это сила самой разной природы, хотя измеряется она в вольтах:

  • Химической. Происходит от процессов химического замещения ионов одних металлов ионами других (более активных). В результате образуются лишние электроны, стремящиеся «спастись» на краю ближайшего проводника. Такой процесс бывает обратимым или необратимым. Обратимый — в аккумуляторах. Их можно зарядить, вернув заряженные ионы обратно в раствор, отчего он приобретет больше, например, кислотности (в кислотных аккумуляторах). Кислотность электролита и есть причина ЭДС аккумулятора, работает непрерывно, пока раствор не станет абсолютно нейтральным химически.

  • Магнитодинамической. Возникает при воздействии на проводник, некоторым образом ориентированный в пространстве, изменяющегося магнитного поля. Или от магнита, движущегося относительно проводника, или от движения проводника относительно магнитного поля. Электроны в этом случае тоже стремятся двигаться в проводнике, что позволяет их улавливать и помещать на выходные контакты устройства, создавая разность потенциалов.

  • Электромагнитной. Переменное магнитное поле создается в магнитном материале переменным электрическим напряжением первичной обмотки. Во вторичной обмотке возникает движение электронов, а значит и напряжение, пропорциональное напряжению в первичной обмотке. Значком ЭДС трансформаторы могут обозначаться в схемах эквивалентного замещения.

  • Фотоэлектрической. Свет, попадая на некоторые проводящие материалы, способен выбивать электроны, то есть делать их свободными. Создается избыток этих частиц, отчего лишние выталкиваются к одному из электродов (аноду). Возникает напряжение, которое и способно породить электрический ток. Такие приборы называются фотоэлементами. Первоначально были придуманы вакуумные фотоэлементы, в которых электроды были установлены в колбе с вакуумом. Электроны в этом случае выталкивались за пределы металлической пластинки (катод), а улавливались другим электродом (анод). Такие фотоэлементы нашли применение в датчиках света. С изобретением же более практичных полупроводниковых фотоэлементов стало возможным создавать из них мощные батареи, чтобы суммированием электродвижущей силы каждого из них вырабатывать существенное напряжение.

  • Теплоэлектрической. Если два разных металла или полупроводника спаять в одной точке, а потом в эту точку доставить тепло, например, свечи, то на противоположных концах пары металлов (термопары) возникает разница в плотностях электронного газа. Эта разница может накапливаться, если соединить термопары последовательной цепочкой, подобно соединению гальванических элементов в батарее или отдельных фотоэлементов в солнечной батарее. ТермоЭДС используется в очень точных датчиках температуры. С этим явлением связано несколько эффектов (Пельтье, Томсона, Зеебека), которые успешно исследуются. Фактом является то, что теплота способна непосредственно превратиться в электродвижущую силу, то есть напряжение.

  • Электростатической. Такие источники ЭДС были придуманы практически одновременно с гальваническими элементами или даже раньше (если считать натирание янтаря шелком нормальным производством ЭДС). Они еще называются электрофорными машинами, или, по имени изобретателя, генераторами Вимшурста. Хотя Вимшурст создал внятное техническое решение, позволяющее снятый потенциал накапливать в лейденской банке — первом конденсаторе (причем, хорошей емкости). Первой же электрофорной машиной можно считать огромный шар из серы, насаженный на ось, — аппарат магдебургского бургомистра Отто фон Герике в середине XVII века. Принцип работы — натирание легко электризующихся от трения материалов. Правда прогресс у фон Герике можно назвать, по поговорке, движимым ленью, когда нет охоты натирать янтарь или что-то другое вручную. Хотя, конечно, этому любознательному политику чего-чего, а фантазии и активности было не занимать. Вспомним хотя бы его же всем известный опыт с разрыванием двумя вереницами ослов (или мулов) шара без воздуха за цепи на два полушария.

Электризация, как первоначально предполагали, происходит именно от «трения», то есть, натирая янтарь тряпкой, мы «срываем» с его поверхности электроны. Однако исследования показали, что здесь не так все просто. Оказывается, на поверхности диэлектриков всегда имеются неравномерности заряда, и к этим неравномерностям притягиваются ионы из воздуха. Образуется такая воздушно-ионная шуба, которую мы и повреждаем, натирая поверхность.

  • Термоэмиссионной. При нагревании металлов с их поверхности срываются электроны. В вакууме они достигают другого электрода и наводят там отрицательный потенциал. Очень перспективное сейчас направление. На рисунке приведена схема защиты гиперзвукового летательного аппарата от перегрева частей корпуса встречным потоком воздуха, причем термоэлектроны, испускаемые катодом (который при этом охлаждается — одновременное действие эффектов Пельтье и/или Томсона), достигают анода, наводя на нем заряд. Заряд, вернее, напряжение, которое равно полученной ЭДС, можно использовать в цепи потребления внутри аппарата.

1 — катод, 2 — анод, 3, 4 — отводы катода и анода, 5 — потребитель

  • Пьезоэлектрической. Многие кристаллические диэлектрики, когда испытывают механическое давление на себя в каком-либо направлении, реагируют на него наведением разницы потенциалов между своими поверхностями. Эта разность зависит от приложенного давления, поэтому уже используется в датчиках давления. Пьезоэлектрические зажигалки для газовых плит не требуют никакого другого источника энергии — только нажатия пальцем на кнопочку. Известны попытки создания пьезоэлектрической системы зажигания в автомобилях на основе пьезокерамики, получающей давление от системы кулачков, связанных с главным валом двигателя. «Хорошие» пьезоэлектрики — у которых пропорциональность ЭДС от давления высоко точна — бывают очень тверды (например, кварц), при механическом давлении почти не деформируются.

  • Однако долгое воздействие давлением на них вызывает их разрушение. В природе мощные слои каменных пород также являются пьезоэлектриками, давления земных толщ наводят громадные заряды на их поверхностях, что порождает в глубинах земли титанические бури и грозы. Однако, не все так страшно.Уже были разработаны и эластичные пьезоэлектрики, и даже уже началось изготовление на их основе (и на основе нанотехнологий) изделий, идущих на продажу.

То, что единицей измерения ЭДС является единица электрического напряжения, понятно. Так как самые разнородные механизмы, создающие электродвижущую силу источника тока, все преобразуют свои виды энергии в движение и накопление электронов, а это в конечном счете и приводит к появлению такого напряжения.

Ток, возникающий от ЭДС

Электродвижущая сила источника тока на то и движущая сила, что электроны от нее начинают двигаться, если замкнуть электрическую цепь. Их к этому принуждает ЭДС, пользуясь своей неэлектрической «половиной» природы, которая не зависит, все-таки, от половины, связанной с электронами. Так как считается, что ток в цепи течет от плюса к минусу (такое определение направления было сделано раньше, чем все узнали, что электрон — отрицательная частица), то внутри прибора с ЭДС ток делает движение завершающее — от минуса к плюсу. И всегда рисуют у знака ЭДС, куда направлена стрелочка – +. Только в обоих случаях — и внутри ЭДС источника тока, и снаружи, то есть в потребляющей цепи, — мы имеем дело с электрическим током со всеми его обязательными свойствами. В проводниках ток наталкивается на их сопротивление. И здесь, в первой половине цикла, имеем сопротивление нагрузки, во второй, внутренней, — сопротивление источника или внутреннее сопротивление.

Внутренний процесс работает не мгновенно (хотя очень быстро), а с определенной интенсивностью. Он совершает работу по доставке зарядов от минуса к плюсу, и это тоже встречает сопротивление…

Сопротивление это двоякого рода.

  1. Внутреннее сопротивление работает против сил, разъединяющих заряды, оно имеет природу, «близкую» этим разъединяющим силам. По крайней мере, работает с ними в едином механизме. Например, кислота, отбирающая кислород у двуокиси свинца и замещающая его на ионы SO 4 -, определенно испытывает некоторое химическое сопротивление. И это как раз и проявляется как работа внутреннего сопротивления аккумулятора.
  2. Когда наружная (выходная) половина цепи не замкнута, появление все новых и новых электронов на одном из полюсов (и убывание их с другого полюса) вызывает усиление напряженности электростатического поля на полюсах аккумулятора и усиление отталкивания между электронами. Что позволяет системе «не идти вразнос» и остановиться на некотором состоянии насыщенности. Больше электронов из аккумулятора наружу не принимается. И это внешне выглядит как наличие постоянного электрического напряжения между клеммами аккумулятора, которое называется U хх, напряжением холостого хода. И оно численно равно ЭДС — электродвижущей силе. Поэтому и единицей измерения ЭДС является вольт (в системе СИ).

Но если только подключить к аккумулятору нагрузку из проводников, имеющих отличное от нуля сопротивление, то немедленно потечет ток, сила которого определяется по закону Ома.

Померить внутреннее сопротивление источника ЭДС, казалось бы, можно. Стоит включить в цепь амперметр и шунтировать (закоротить) внешнее сопротивление. Однако внутреннее сопротивление настолько низко, что аккумулятор начнет разряжаться катастрофически, вырабатывая огромное количество теплоты, как на внешних закороченных проводниках, так и во внутреннем пространстве источника.

Однако можно поступить иначе:

  1. Измерить E (помним, напряжение холостого хода, единица измерения — вольт).
  2. Подключить в качестве нагрузки некоторый резистор и померить падение напряжения на нем. Вычислить ток I 1 .
  3. Вычислить значение внутреннего сопротивления источника ЭДС можно, воспользовавшись выражением для r

Обычно способность аккумулятора выдавать электроэнергию оценивается его энергетической «емкостью» в амперчасах. Но интересно было бы посмотреть, какой максимальный ток он может вырабатывать. Несмотря на то, что, быть может, электродвижущая сила источника тока заставит его взорваться. Так как идея устроить на нем короткое замыкание показалась не очень заманчивой, можно вычислить эту величину чисто теоретически. ЭДС равно U хх. Просто нужно дорисовать график зависимости падения напряжения на резисторе от тока (следовательно, и от сопротивления нагрузки) до точки, в которой сопротивление нагрузки будет равно нулю. Это точка I кз , пересечения красной линии с линией координаты I , в которой напряжение U стало нулевым, а все напряжение E источника будет падать на внутреннее сопротивление.

Часто кажущие простыми основные понятия не всегда бывает можно понять без привлечения примеров и аналогий. Что такое электродвижущая сила, и как она работает, можно представить, только рассмотрев множество ее проявлений. А стоит рассмотреть определение ЭДС, как оно дается солидными источниками посредством умных академических слов — и все начинай с начала: электродвижущая сила источника тока. Или просто выбей на стене золотыми буквами:

Выясним, какая величина является основной характеристикой источника тока. Любой источник тока имеет два полюса: положительный и отрицательный. Чтобы он имел эти полюсы, необходимо внутри его собрать свободные положительные заряды на одном полюсе, а отрицательные — на другом. Для этого надо совершить работу. Эту работу не могут совершить электростатические силы, так как разноименные заряды притягиваются, а их надо разъединить. Работа по накоплению зарядов производится не электростатическими силами, а сторонними. Природа последних может быть различна. Например, в генераторах электрического тока разделение зарядов осуществляется силам магнитного поля, в аккумуляторах и гальванических элементах — химическими. Исследование источников тока показывает, что отношение работы сторонней силы к заряду, накопленному на полюсе, для данного источника тока есть величина постоянная и называется электродвижущей силой источника тока:

Электродвижущая сила источника тока

Скалярная величина, являющаяся характеристикой источника тока и измеряемая работой, совершенной сторонней силой внутри его по накоплению на каждом полюсе по 1 к заряда, называется электродвижущей силой источника тока. Заряд в 1 к , накопленный на полюсе источника тока, обладает потенциальной электрической энергией, численно равной э. д. с. источника.

Единица э. д. с.

Замерим э. д. с. источника тока. К демонстрационному гальваническому элементу подключим вольтметр (рис. 75, а) * . Меняя взаимное расположение электродов в электролите, а также величину погружения их в электролит, видим, что показания вольтметра (1,02 в ) не изменяются. Э. д. с. не зависит от размера источника тока. Она зависит только от природы сторонних сил, вызывающих накопление зарядов на полюсах. Каждый источник тока имеет свою э. д. с.

* (При таком замере э. д. с. показание вольтметра будет немного меньше, чем величина э. д. с. Чем больше сопротивление катушки вольтметра по сравнению с внутренним сопротивлением источника, тем меньше будет эта разница, что и наблюдается в описываемом опыте. )

При замыкании электрической цепи источник тока образует в проводах стационарное электрическое поле и передает ему энергию, накопленную зарядами на его полюсах. За счет этой энергии стационарное поле совершает работу на образование тока, передавая ему свою энергию, которую потребитель тока преобразует в другие виды энергии.

Внутренняя часть цепи, которую составляет источник тока, как и любой проводник, обладает сопротивлением; оно называется внутренним сопротивлением источника тока r . У генератора тока внутренним сопротивлением является сопротивление обмотки якоря, у химических источников сопротивление электролита.

При замыкании цепи электрическое поле, перемещая заряд 1 к из точки А в точку В по внешнему участку цепи (рис. 75, б), совершает работу, которая численно равна напряжению U на этом участке. Достигнув полюса В, заряд 1 к должен перейти на внутренний участок цепи и переместиться на полюс А. Чтобы он снова оказался на полюсе А и имел такую же энергию Е, как и при выходе из точки А, над ним сторонние силы источника тока должны совершить работу, равную работе, затраченной на его перемещение по внешнему участку цепи, которая численно равна напряжению U на этом участке, плюс работа, затраченная на преодоление внутреннего сопротивления r источника. Последняя численно равна напряжению u на внутреннем участке цепи. Следовательно, э. д. с. источника численно равна Е = U + u. Электродвижущая сила численно равна работе, которую источник тока совершает, перемещая заряд 1 к по всей цепи .

Замерим напряжение на внешнем и внутреннем участках; цепи (рис. 75, в) * . Вольтметр А показывает напряжение на внешнем сопротивлении R, а вольтметр В — на внутреннем; сопротивлении r. Меняя величину сопротивления внешней цепи; замечаем, что при этом изменяется напряжение на участках цепи (табл. 4).

* (Щупы 1 и 2 изготовлены из толстого медного провода в хлорвиниловой изоляции, которая срезана со стороны, расположенной к середине сосуда. Щупы соприкасаются изоляцией с электродами. )

Видим, что сумма напряжений на внешнем и внутреннем участках цепи — величина постоянная (в пределах погрешностей опыта) и равна э. д. с. источника. Она показывает величину той энергии, которую источник тока в состоянии передать в электрическую цепь при перемещении по всей цепи заряда в 1 к.

Закон Ома для замкнутой цепи

Закон Ома для замкнутой цепи показывает — значение тока в реальной цепи зависит не только от сопротивления нагрузки, но и от сопротивления источника.

Формулировка закона Ома для замкнутой цепи звучит следующим образом: величина тока в замкнутой цепи, состоящей из источника тока, обладающего внутренним и внешним нагрузочным сопротивлениями, равна отношению электродвижущей силы источника к сумме внутреннего и внешнего сопротивлений.

Впервые зависимость тока от сопротивлений была экспериментально установлена и описана Георгом Омом в 1826 году.

Формула закона Ома для замкнутой цепи записывается в следующем виде:

где:

  • I [А] – сила тока в цепи,
  • ε [В] – ЭДС источника напряжения,
  • R [Ом] – сопротивление всех внешних элементов цепи,
  • r [Ом] – внутреннее сопротивление источника напряжения

Физический смысл закона

Потребители электрического тока вместе с источником тока образуют замкнутую электрическую цепь. Ток, проходящий через потребитель, проходит и через источник тока, а значит, току кроме сопротивления проводника оказывается сопротивление самого источника. Таким образом, общее сопротивление замкнутой цепи будет складываться из сопротивления потребителя и сопротивления источника.

Физический смысл зависимости тока от ЭДС источника и сопротивления цепи заключается в том, что чем больше ЭДС, тем больше энергия носителей зарядов, а значит больше скорость их упорядоченного движения. При увеличении сопротивления цепи энергия и скорость движения носителей зарядов, следовательно, и величина тока уменьшаются.

Зависимость можно показать на опыте. Рассмотрим цепь, состоящую из источника, реостата и амперметра. После включения в цепи идет ток, наблюдаемый по амперметру, двигая ползунок реостата, увидим, что при изменении внешнего сопротивления ток будет меняться.

Примеры задач на применение закона Ома для замкнутой цепи

К источнику ЭДС 10 В и внутренним сопротивлением 1 Ом подключен реостат, сопротивление которого 4 Ом. Найти силу тока в цепи и напряжение на зажимах источника.

Дано:Решение:
  • ε = 10 В
  • r = 1 Ом
  • R = 4 Ом
  • Запишем закон Ома для замкнутой цепи — I=ε/(R+r) .
  • Падение напряжения на зажимах источника найдем по формуле U=ε-Ir=εR/(R+r).
  • Подставим заданные значения и вычислим I=(10 В)/((4+1)Ом)=2 А, U=(10 В∙4Ом)/(4+1)Ом=8 В./li>
  • Ответ: 2 А, 8 В.

При подключении к батарее гальванических элементов резистора сопротивлением 20 Ом сила тока в цепи была 1 А, а при подключении резистора сопротивлением 10 Ом сила тока стала 1,5 А. Найти ЭДС и внутреннее сопротивление батареи.

Дано:Решение:
  • R1 = 20 Ом
  • R2 = 10 Ом
  • I1 = 1 A
  • I2 = 1.5 A
  • Запишем закон Ома для замкнутой цепи — I=ε/(R+r) .
  • Отсюда для каждого сопротивления получим ε=I_1 R_1+I_1 r, ε=I_2 R_2+I_2 r. .
  • Приравняем правые части уравнений и найдем внутреннее сопротивление r=(I_1 R_1-I_2 R_2)/(I_2-I_1 ).
  • Подставим полученное значение в закон Ома ε=(I_1 I_2 (R_2-R_1))/(I_2-I_1 ).
  • Произведем вычисления r=(1А∙20 Ом-1,5А∙10Ом)/(1,5-1)А=10 Ом, ε=(1А∙1,5А(20-10)Ом)/((1,5-1)А)=30 В.
  • Ответ: 30 В, 10 Ом.

ЭДС аккумулятора

просмотров 8 733 Google+

ЭДС аккумулятора

ЭДС аккумулятора (Электродвижущая сила) это разность электродных потенциалов при отсутствии внешней цепи. Электродный потенциал складывается из равновесного электродного потенциала. Он характеризует состояние электрода в состоянии покоя, то есть отсутствии электрохимических процессов, и потенциала поляризации, определяющемуся как разность потенциалов электрода при зарядке (разрядке) и при отсутствии цепи.

Процесс диффузии.

Благодаря процессу диффузии, выравниванию плотности электролита в полости корпуса аккумулятора и в порах активной массы пластин, электродная поляризация может сохраняться в аккумуляторе при отключении внешней цепи.

Скорость прохождения диффузии напрямую зависит от температуры электролита, чем выше температура, тем быстрее проходит процесс и может сильно отличаться по времени, от двух часов до суток. Наличие двух составляющих электродного потенциала при переходных режимах привело к разделению на равновесную и не равновесную ЭДС аккумулятора.
На равновесную ЭДС аккумулятора влияет содержание и концентрация ионов активных веществ в электролите, а так же химические и физические свойства активных веществ. Главную роль в величине ЭДС играет плотность электролита. Так как  плотность электролита зависит от температуры, то и ЭДС так же зависит от температуры. Зависимость ЭДС от плотности можно выразить формулой:

Е = 0,84 + р Где Е – ЭДС аккумулятора (В) Р – плотность электролита приведённая к температуре 25 гр. С (г/см3)Эта формула истинна при рабочей плотности электролита в пределах 1,05 – 1,30 г/см3. ЭДС не может характеризовать степень разрежённости аккумулятора напрямую. Но если замерить его на выводах и сравнить с расчётным по плотности, то можно, с долей вероятности, судить о состоянии пластин и ёмкости.
В состоянии покоя плотность электролита в порах электродов и полости моноблока одинаковы и равны ЭДС покоя. При подключении потребителей или источника заряда, изменяется поляризация пластин и концентрация электролита в порах электродов. Это приводит к изменению ЭДС. При заряде значение ЭДС увеличивается, а при разряде уменьшается. Это связано с изменением плотности электролита, который участвует в электрохимических процессах.

ЭДС аккумулятора не равна напряжению аккумулятора которое зависит от наличия или отсутствия нагрузки на его клеммах.

admin 25/07/2011 «Если Вы заметили ошибку в тексте, пожалуйста выделите это место мышкой и нажмите CTRL+ENTER» «Если статья была Вам полезна, поделитесь ссылкой на неё в соцсетях»

Несколько источников ЭДС в цепи

Если в цепи присутствует несколько ЭДС подключенных последовательно, то:

1. При правильном (положительный полюс одного источника присоединяется к отрицательному другого) подключении источников общее ЭДС всех источников и их внутреннее сопротивление может быть найдено по формулам:

Например, такое подключение источников осуществляется в пультах дистанционного управления, фотоаппаратах и других бытовых приборах, работающих от нескольких батареек.

2. При неправильном (источники соединяются одинаковыми полюсами) подключении источников их общее ЭДС и сопротивление рассчитывается по формулам:

В обоих случаях общее сопротивление источников увеличивается.

При параллельном подключении имеет смысл соединять источники только c одинаковой ЭДС, иначе источники будут разряжаться друг на друга. Таким образом суммарное ЭДС будет таким же, как и ЭДС каждого источника, то есть при параллельном соединении мы не получим батарею с большим ЭДС. При этом уменьшается внутреннее сопротивление батареи источников, что позволяет получать большую силу тока и мощность в цепи:

В этом и состоит смысл параллельного соединения источников. В любом случае при решении задач сначала надо найти суммарную ЭДС и полное внутреннее сопротивление получившегося источника, а затем записать закон Ома для полной цепи.

 

Работа и мощность тока. Закон Джоуля-Ленца

К оглавлению…

РаботаAэлектрического тока I, протекающего по неподвижному проводнику с сопротивлением R, преобразуется в теплоту Q, выделяющееся на проводнике. Эту работу можно рассчитать по одной из формул (с учетом закона Ома все они следуют друг из друга):



Закон преобразования работы тока в тепло был экспериментально установлен независимо друг от друга Дж.Джоулем и Э.Ленцем и носит названиезакона Джоуля–Ленца. Мощность электрического тока равна отношению работы тока A к интервалу времени Δt, за которое эта работа была совершена, поэтому она может быть рассчитана по следующим формулам:

Работа электрического тока в СИ, как обычно, выражается в джоулях (Дж), мощность – в ваттах (Вт).

 

Энергобаланс замкнутой цепи

К оглавлению…

Рассмотрим теперь полную цепь постоянного тока, состоящую из источника с электродвижущей силой ε и внутренним сопротивлением r и внешнего однородного участка с сопротивлением R. В этом случае полезная мощность или мощность, выделяемая во внешней цепи:

Максимально возможная полезная мощность источника достигается, если R = r и равна:

Если при подключении к одному и тому же источнику тока разных сопротивлений R1 и R2 на них выделяются равные мощности то внутреннее сопротивление этого источника тока может быть найдено по формуле:

Мощность потерь или мощность внутри источника тока:

Полная мощность, развиваемая источником тока:

КПД источника тока:

 

Электролиз

К оглавлению…

Электролитами принято называть проводящие среды, в которых протекание электрического тока сопровождается переносом вещества. Носителями свободных зарядов в электролитах являются положительно и отрицательно заряженные ионы. К электролитам относятся многие соединения металлов с металлоидами в расплавленном состоянии, а также некоторые твердые вещества. Однако основными представителями электролитов, широко используемыми в технике, являются водные растворы неорганических кислот, солей и оснований.

Прохождение электрического тока через электролит сопровождается выделением вещества на электродах. Это явление получило название электролиза.

Электрический ток в электролитах представляет собой перемещение ионов обоих знаков в противоположных направлениях. Положительные ионы движутся к отрицательному электроду (катоду), отрицательные ионы – к положительному электроду (аноду). Ионы обоих знаков появляются в водных растворах солей, кислот и щелочей в результате расщепления части нейтральных молекул. Это явление называется электролитической диссоциацией.

Закон электролиза был экспериментально установлен английским физиком М.Фарадеем в 1833 году. Закон Фарадея определяет количества первичных продуктов, выделяющихся на электродах при электролизе. Итак, масса m вещества, выделившегося на электроде, прямо пропорциональна заряду Q, прошедшему через электролит:

Величину k называют электрохимическим эквивалентом. Он может быть рассчитан по формуле:

где: n – валентность вещества, NA – постоянная Авогадро, M – молярная масса вещества, е – элементарный заряд. Иногда также вводят следующее обозначение для постоянной Фарадея:

 


АБИТУРИЕНТУ

АБИТУРИЕНТУ
С.С.Чесноков, Г.Я.Мякишев, С.Ю.Никитин,
Н.Б.Подымова, М.С.Полякова, В.И.Шмальгаузен,
физический факультет МГУ, г. Москва

Задачи, предлагавшиеся на устных вступительных экзаменах
на факультет вычислительной математики и кибернетики МГУ им. М.В.Ломоносова в 1999 г.

III. ЭЛЕКТРОДИНАМИКА

1 Заряженная частица влетает в пространство между пластинами плоского конденсатора, напряжение на котором поддерживается постоянным. Начальная скорость частицы параллельна пластинам конденсатора. При вылете из конденсатора скорость частицы составила угол a = 60° с ее начальной скоростью. Под каким углом b к начальной скорости вылетит эта частица из конденсатора, если расстояние между его пластинами увеличить в k = 3 раза? Силу тяжести не учитывать.

Решение

Определим вначале, как зависит угол вылета частицы из конденсатора от ее заряда q, массы m и начальной скорости v0, а также от параметров электрической системы. Обозначим через U напряжение на зажимах источника, L – длину пластин, d0 – начальное расстояние между ними. В пространстве между пластинами на частицу действует сила F = qE, направленная вдоль линий напряженности электрического поля E в конденсаторе. В результате движение частицы в направлении, параллельном пластинам, будет равномерным со скоростью v0, а в направлении, перпендикулярном пластинам, – равноускоренным с ускорением

За время пролета частицей конденсатора перпендикулярная пластинам составляющая скорости частицы приобретет значение: 

Следовательно, угол a между начальной скоростью частицы и ее скоростью при вылете из конденсатора определится из соотношения:

При увеличении расстояния между пластинами в k раз угол вылета частицы из конденсатора выразится формулой:

Сравнивая два последние соотношения, получаем ответ:

2 Батарея с ЭДС = 2 В и внутренним сопротивлением r = 0,1 Ом присоединена к цепи, изображенной на рисунке. Сопротивление каждого из резисторов R = 1 Ом. Найдите напряжение на клеммах батареи. Сопротивлением всех соединительных проводов пренебречь.

 Решение

 Для того чтобы определить напряжение на клеммах батареи, необходимо вычислить сопротивление нагрузки. Нетрудно заметить, что потенциалы точек А и С, а также точек В и D попарно равны. Следовательно, все три резистора нагрузки фактически соединены параллельно. Поэтому сопротивление внешней цепи Rвнеш= R/3, сила тока в цепи Отсюда получаем ответ:

3 Во внешней нагрузке, подключенной к батарее, выделяется мощность W1 = 1 Вт. Чему равен коэффициент полезного действия h этой цепи (т.е. отношение мощности, выделяющейся в нагрузке, к полной мощности, развиваемой батареей), если при подключении той же нагрузки к двум таким батареям, соединенным последовательно, мощность в нагрузке стала равной W2 = 1,44 Вт?

Решение

В цепи, состоящей из батареи и внешней нагрузки сопротивлением R, мощность, выделяющаяся в нагрузке, равна:

где I – ток в цепи, – ЭДС батареи, r – ее внутреннее сопротивление. При этом полная мощность, развиваемая батареей:

Отсюда следует, что коэффициент полезного действия цепи:

Если подключить эту же нагрузку к двум одинаковым батареям, соединенным последовательно, ЭДС и внутреннее сопротивление в цепи станут равными соответственно
2 и 2r. Следовательно, мощность, выделяющаяся в нагрузке в этом случае, будет равна:

. Составим отношение:

Отсюда получаем ответ:

4 При подключении нагрузки к батарее с внутренним сопротивлением r1 = 0,1 Ом во внешней цепи выделяется мощность W1 = 1 Вт. В той же нагрузке, питаемой от батареи внутренним сопротивлением r2 = 0,2 Ом и c равной ЭДС, выделяется мощность W2 = 0,64 Вт. Чему равно сопротивление нагрузки R?

Решение

Мощность, выделяющаяся в нагрузке сопротивлением R, подключенной к батарее с ЭДС и внутренним сопротивлением r1, равна При подключении этой нагрузки к батарее с той же ЭДС, но внутренним сопротивлением r2, в ней будет выделяться мощность Составим отношение:

Выражая из последнего соотношения R, получаем ответ:

5 При подключении к аккумулятору внутренним сопротивлением r = 0,16 Ом нагревательный элемент потребляет мощность W1 = 200 Вт. При подключении этого же нагревательного элемента к двум таким аккумуляторам, соединенным последовательно, выделяемая в нем мощность составляет W2 = 288 Вт. Найдите ЭДС аккумулятора.

Решение

Мощность, выделяемая в нагревательном элементе при подключении его к одному аккумулятору, равна где R – сопротивление нагревателя. При подключении этого же элемента к двум одинаковым аккумуляторам, соединенным последовательно, значения ЭДС и внутреннего сопротивления удваиваются, и в нагревателе выделяется мощность: 

Вводя величину  получим  Отсюда 

Учитывая, что  после несложных преобразований получаем ответ:

6 Электрон влетает в однородное электрическое поле напряженностью E = 6 ·104 В/м перпендикулярно силовым линиям. Определите величину и направление вектора индукции магнитного поля B, которое надо создать в этой области пространства для того, чтобы электрон пролетел ее, не отклоняясь от первоначального направления. Кинетическая энергия электрона EK=1,6•10-16 Дж, масса электрона m = 9 · 10–31 кг. Силой тяжести пренебречь.

 Решение

В данном однородном электрическом поле напряженностью E электрон движется под действием кулоновской силы FК по параболической траектории (здесь |e| – абсолютная величина заряда электрона).

Для того чтобы электрон двигался прямолинейно, нужно создать такое однородное магнитное поле, в котором действующая на электрон сила Лоренца FЛ в каждой точке его траектории была бы равна кулоновской силе по величине и противоположна ей по направлению, т.е. FЛ = –FК. Величина силы Лоренца  (здесь v0 – скорость электрона, – величина составляющей вектора магнитной индукции, перпендикулярной скорости), ее направление определяется правилом левой руки. Очевидно, что сила Лоренца направлена против кулоновской силы в том случае, если магнитная индукция направлена перпендикулярно начальной скорости электрона и напряженности электрического поля. Применяя правило левой руки, с учетом того, что заряд электрона отрицателен, находим, что вектор магнитной индукции должен быть перпендикулярным плоскости рисунка и направлен на нас. Составляя равенство |e| E=|e| v0E и учитывая, что получаем ответ:

Как измерить выходную мощность от батареи

Батареи выдают мощность, когда они подключены к цепи. Батарея, которая не подключена к цепи, не дает тока и, следовательно, не выводит энергию. Однако после того, как вы подключили батарею к цепи, вы можете определить выходную мощность, измерив падение напряжения на нагрузке цепи. Если вы знакомы с уравнениями, связывающими мощность с напряжением, током и сопротивлением, вы сможете легко перемещаться между этими понятиями.

Расчет мощности

Мощность — это произведение тока и напряжения. Следовательно, чтобы рассчитать выходную мощность батареи, вы должны измерить эти два аспекта цепи. Ток — это поток заряда в единицу времени, а напряжение — это электрическая потенциальная энергия. Единицами измерения тока и напряжения являются амперы и вольт соответственно. Кроме того, напряжение — это произведение тока и сопротивления. Сопротивление — это величина сопротивления току.Предметы, подверженные электрическому потенциалу или напряжению, обладают характеристическим сопротивлением. Сопротивление измеряется в Ом. Из-за взаимосвязанного характера мощности, напряжения, тока и сопротивления вы можете определить мощность, даже если вам известны только две другие величины. Например, мощность равна квадрату тока, умноженному на сопротивление, или квадрату напряжения, деленному на сопротивление.

Проведение измерений

Чтобы измерить выходную мощность батареи, вы должны измерить ее, когда она подключена к внешнему сопротивлению, также называемому сопротивлением нагрузки.В противном случае аккумулятор не будет работать и, следовательно, не будет обеспечивать выходную мощность. Сопротивление нагрузки создает измеримое падение напряжения. Если вы знаете сопротивление нагрузки, вы можете определить ток. Используйте мультиметр, чтобы проверить падение напряжения на нагрузке. Поверните шкалу мультиметра, чтобы контролировать напряжение постоянного тока. Затем поместите два провода измерителя по обе стороны от нагрузки. Полярность значения не имеет. Разделите это напряжение на сопротивление нагрузки, чтобы получить ток. Как только у вас есть ток и напряжение, умножьте их, чтобы получить выходную мощность.Вы заметите, что выходная мощность батареи зависит от схемы, в которой она запитана. Это связано с тем, что ток изменяется в зависимости от сопротивления нагрузки.

Напряжение замкнутой и разомкнутой цепи

Напряжение батареи изменяется в зависимости от того, приложено ли оно к цепи. Подробное описание батареи часто включает цифры напряжения как в замкнутой, так и в разомкнутой схемах. Напряжение замкнутой цепи аккумулятора также называется напряжением на его клеммах.Кроме того, эти напряжения могут варьироваться в зависимости от состояния заряда аккумулятора и тока зарядки, где это применимо. Это еще одна причина, по которой вы должны измерять напряжение батареи, когда она подключена к цепи.

Внутреннее сопротивление

Батарея показывает внутреннее сопротивление в дополнение к любому сопротивлению в цепи. По мере увеличения этого внутреннего сопротивления выходная мощность батареи уменьшается, поскольку некоторая мощность рассеивается внутри. Когда это происходит, напряжение на клеммах аккумулятора уменьшается.Если внутреннее сопротивление батареи становится слишком высоким, батарея больше не будет обеспечивать достаточный потенциал для пропускания тока через нагрузку.

Номинальные характеристики аккумуляторов

Аккумуляторы имеют несколько напечатанных номинальных значений их емкости и мощности. Общий электростатический потенциал батареи указывается в вольтах. Это одна из самых выдающихся характеристик батареи, и она имеет большое значение для выходной мощности батареи: как правило, чем выше номинальное напряжение, тем больше выходная мощность.Также емкость аккумулятора указана в ампер-часах. Это выражение количества ампер, которое батарея будет выдавать в течение заданного количества часов. Например, батарея на 140 ампер-час может выдавать ток 7 ампер в течение 20 часов, прежде чем потребуется зарядка.

Формула ЭДС — уравнения, решаемые примеры и часто задаваемые вопросы

Электродвижущая сила — одно из важных понятий, которые помогают нам понять процесс электромагнетизма. Электродвижущая сила сокращенно называется ЭДС и тесно связана с более распространенным понятием напряжения.Электродвижущая сила — это полная энергия, обеспечиваемая батареей или элементом на кулон q заряда, проходящего через нее.

Величина ЭДС равна разности потенциалов на клеммах ячейки, когда нет тока, протекающего через данную электрическую цепь, и используемая формула известна как формула ЭДС. В этой статье мы подробно изучим формулу электродвижущей силы, идею физики ЭДС и, наконец, с помощью уравнения ЭДС с несколькими решенными примерами.

EMF Physics

Электродвижущая сила может быть определена как полное напряжение или разность потенциалов на клеммах батареи в разомкнутой цепи или, другими словами, когда через нее не течет ток. Это может выглядеть не так, поскольку это будет отличаться, но каждая батарея будет построена с определенным внутренним сопротивлением. Это связано с регулярным сопротивлением, которое уменьшает ток в электрической цепи, но оно заключено только внутри самой батареи.

Мы знаем, что когда цепь разомкнута, ток не будет течь через элемент, это означает, что внутреннее сопротивление батареи ничего не изменит, потому что нет тока, который мог бы уменьшить или замедлить. Таким образом, электродвижущую силу можно рассматривать как максимальную разность потенциалов или напряжение на двух выводах в идеализированном состоянии. Это объясняет физику ЭДС, и из этого мы можем понять, что электродвижущая сила — это частный случай разности напряжений.

Теперь возник вопрос, хотя электродвижущая сила вообще не является формой силы, тогда почему ее называют электродвижущей силой, в чем разница между ЭДС и регулярной разностью потенциалов и что будет источник ЭДС? Чтобы ответить на эти сомнения, рассмотрим простую электрическую схему лампы, подключенной к батарее.

Мы знаем, что любое гальваническое устройство можно представить как двухполюсное устройство, которое поддерживает один вывод с более высоким потенциалом, а другой вывод — с более низким потенциалом.Более высокий электрический потенциал обычно известен как положительный полюс и обычно обозначается знаком плюс. Клемма с более низким потенциалом называется отрицательной клеммой и обозначается знаком минус. Это называется источником ЭДС.

Когда источник электродвижущей силы отключен от лампы, т. Е. Когда цепь разомкнута, то в данном источнике ЭДС нет чистого движения зарядов. После того, как цепь будет замкнута или повторно подключена к лампе, заряды будут перемещаться от одной клеммы батареи через лампу, что в дальнейшем приведет к тому, что лампа начнет светиться и вернется к другой клемме батареи.

Если мы рассмотрим обычный поток электрического тока, то есть положительный ток, положительные заряды имеют тенденцию покидать положительный вывод, проходить через лампу и попадать на отрицательный конец источника ЭДС. Так устроен источник ЭМП. В то же время электродвижущая сила батареи — это разность потенциалов, развиваемая на обоих концах данной батареи.

Таким образом, физика ЭДС объясняет, что электродвижущая сила — это полная энергия, поставляемая батареей или элементом на кулон заряда, проходящего через нее.Общая величина ЭДС равна напряжению или разности потенциалов на клеммах батареи, когда в данной электрической цепи нет тока.

Уравнение ЭДС

Мы знаем, что заряды циркулируют в электрической цепи, для движения зарядов в данной электрической цепи нам необходимо приложить к ней внешнюю силу. Мы говорим, что внешний источник электричества, такой как батарея, использует такую ​​силу, которая придает ускорение зарядам, и это называется электродвижущей силой.Несмотря на название, это не форма силы, а разность потенциалов, на самом деле, это особый случай разности потенциалов, который обычно обозначается символом.

Теперь давайте посмотрим на уравнение ЭДС:

Согласно определению физики ЭДС и ЭДС формула ЭДС имеет вид:

\ [\ Rightarrow EMF = \ varepsilon = \ frac {E} { Q} \]… .. (1)

Где,

E — Полная энергия батареи

Q — Полный заряд, протекающий через данный контур

Уравнение (1) можно использовать, если мы знаем общую энергия батареи, используемой в цепи.Электродвижущая сила — это также разность потенциалов, возникающая в цепи, поэтому формулу ЭДС можно также найти с помощью закона Ома. Следовательно, мы пишем:

ε = IR… .. (2)

Где,

I — полный ток, протекающий в цепи

R — полное сопротивление, используемое в цепи

Поскольку мы знаем, что ЭДС зависит от при внутреннем сопротивлении батареи мы должны заменить сопротивление суммой сопротивления и внутреннего сопротивления.Таким образом, уравнение (2) принимает следующий вид:

ε = I (r + R)

ε = Ir + IR

ε = V + Ir ……. (3)

Где,

В — Общая разность потенциалов в цепи

I — Полный ток, протекающий в цепи

r — Внутреннее сопротивление батареи

Следовательно, уравнение (1) и уравнение (3) известны как формула ЭДС или уравнение ЭДС. Давайте разберемся с формулой ЭДС и как найти ЭДС или как рассчитать ЭДС на нескольких решенных примерах.

Примеры:

1. Рассмотрим электрическую цепь с разностью потенциалов 5 В с током 1 А. Если внутреннее сопротивление используемой батареи составляет 0,8 Ом. Затем определите ЭДС цепи, используя формулу ЭДС.

Sol:

Дано,

Разность потенциалов электрической цепи = V = 5 вольт

Суммарный ток, протекающий по цепи = I = 1 A

Внутреннее сопротивление батареи = r = 0,8 Ом

Нас просят определить ЭДС цепи, используя уравнение ЭДС.Мы знаем, что ЭДС цепи можно рассчитать по формуле, приведенной ниже:

ε = V + Ir ……. (1)

Где,

В — Полная разность потенциалов, развиваемая в цепи

I — Суммарный ток, протекающий в цепи

r — Внутреннее сопротивление батареи

Подставляя значение разности потенциалов, тока и внутреннего сопротивления в уравнение (1), получаем:

ε = V + Ir

ε = 5 + (10,8)

ε = 5.8 Вольт

Следовательно, ЭДС схемы по формуле ЭДС составляет 5,8 Вольт.

2. Рассчитайте разность потенциалов на клеммах батареи, когда она подключена к нагрузке 10 Ом с ЭДС батареи, ε = 3 вольта и внутреннее сопротивление батареи 2 Ом.

Sol:

Дано,

Суммарная ЭДС батареи = ε = 3 вольта

Внешняя нагрузка, приложенная к батарее = R L = 10 Ом

Внутреннее сопротивление батареи = r = 2 Ом

Теперь нас просят определить разность потенциалов на клеммах аккумулятора.Перед этим рассчитаем ток, протекающий по данной цепи. согласно закону Ома мы знаем, что:

\ [\ Rightarrow I = \ frac {V} {R} = \ frac {\ varepsilon} {r + R_ {L}} \]…. (1)

I = 3/12 = 0,25 А

Теперь давайте определим разность потенциалов на клеммах аккумулятора. Формула ЭДС определяется следующим образом:

ε = V + Ir ……. (2)

Где,

V — полная разность потенциалов, развиваемая в цепи

I — полный ток, протекающий в цепи

r — Внутреннее сопротивление аккумулятора

Подставляя значение ЭДС, тока и внутреннего сопротивления в уравнение (2), получаем:

3 = V + (0.252)

В = 3 — 0,5

В = 2,5 В

Следовательно, разность потенциалов на клеммах аккумулятора составляет 2,5 В.

ячеек последовательно и параллельно — Учебный материал для IIT JEE


Начало ячеек

Алессандро Вольта изобрел электрическую батарею. Впервые он был назван Voltaic Pile . За его вклад в науку единица электрического потенциала получила название Вольт .Джон Фредерик Даниэлл разработал ячейку Даниэля. Затем Джордж Лекланш изобрел влажную батарею, а доктор Карл Гасснер представил сухую батарею. Гастон Планте представил первую аккумуляторную батарею. Это свинцово-кислотная аккумуляторная батарея, которая снова наиболее часто используется в автомобилях.

Ячейка

Мы знаем, что электрический ток — это поток заряженных частиц. Это поток электронов по цепи.

Набор из двух или более ячеек, соединенных последовательно, называется батареей A .Батарея — это источник энергии, преобразующий химическую энергию в электрическую. Он также известен как электрохимическая ячейка . Энергия хранится в форме химической энергии внутри батареи. Аккумуляторы дают нам удобный источник энергии для питания устройств без кабелей и проводов. Когда он подключен к цепи, он производит электрическую энергию. Батарея состоит из двух клемм — положительной и отрицательной. Положительный вывод называется катодом , а отрицательный вывод называется анодом .Их также называют электродами ячейки . Эти электроды будут погружены в раствор под названием Электролит . Это жидкость, которая является ионной и проводит электричество.

Когда аккумулятор собирается заряжаться, к нему подключается внешний источник. Анод батареи подключен к отрицательной клемме источника, а катод — к положительной клемме источника. Поскольку внешний источник подключен к батарее, электроны вставляются в анод.Когда элемент или батарея подключены к цепи, происходят химические реакции. Таким образом, химические реакции происходят внутри двух электродов. Здесь происходят реакции окисления и восстановления. Затем на катоде происходит реакция восстановления, а на аноде — процесс окисления.

Сухая камера

Катод действует как окислитель, принимая электроны от отрицательного концевого анода. Анод действует как восстановитель, теряя электроны.Таким образом, из-за этих химических реакций возникает электрическая разница между выводами-анодом и катодом. При отсутствии питания электролит запрещает движение электронов непосредственно от анода к катоду. Вот почему мы используем внешний источник или подключаемся к цепи. Таким образом, электроны перемещаются от анода к катоду, когда цепь замкнута. Наконец, он дает питание подключенному к нему прибору. Спустя долгое время, когда электрохимический процесс изменяет материалы анода и катода, он перестает выделять электроны.Потом садится аккум.

ЭДС:

ЭДС или электродвижущая сила определяется как разность потенциалов, которая возникает между двумя выводами батареи в разомкнутой цепи. Мы знаем, что анод имеет положительный потенциал (V + ), а катод имеет отрицательный потенциал (V ). Таким образом, ЭДС — это разность потенциалов между анодом положительного вывода и катодом отрицательного вывода, когда через него не протекает ток. ЭДС измеряет энергию, которая передается заряду, переносимому в элементе или батарее.Это энергия в джоулях, деленная на заряд в кулонах. ЭДС действует как инициирующая сила для протекания тока.

ε = E / Q, где ε — электродвижущая сила, E — энергия, а Q — заряд.

ЭДС, которая обозначается ε, а уравнение задается формулой ε = V + — (-V ) = V + + V -. Измеряется в вольтах.

Внутреннее сопротивление:

Внутреннее сопротивление — это сопротивление внутри батареи, которое препятствует протеканию тока при подключении к цепи.Таким образом, он вызывает падение напряжения, когда через него протекает ток. Это сопротивление, обеспечиваемое электролитом и электродами, присутствующими в элементе. Таким образом, внутреннее сопротивление обеспечивается электродами и электролитом, которые препятствуют прохождению тока внутри ячейки.

Уравнение ЭДС и внутреннего сопротивления:

Рассмотрим схему, приведенную ниже. Ячейку можно модифицировать с помощью ЭДС ε и внутреннего резистора с сопротивлением r, включенного последовательно. Внешний нагрузочный резистор с сопротивлением R также подключен к цепи.Разность потенциалов на клеммах, представленная как V, определяется как разность потенциалов, возникающая между положительной и отрицательной клеммами ячейки, когда ток течет по цепи.

V = V + + V — Ir. Это падение напряжения, вызванное внутренним сопротивлением.

Мы знаем, что ε = V + + V -. = Я (R + r).

ε = ИК + Ir.

= V + Ir

В = ε — Ir.

Итак, V = ε — Ir, где V — разность потенциалов в цепи, ε — ЭДС, I — ток, протекающий по цепи, r — внутреннее сопротивление.

Обычно внутреннее сопротивление ячейки не учитывается, потому что ε >> Ir. Величина внутреннего сопротивления меняется от ячейки к ячейке.


Последовательные и параллельные соединения

В основном есть два типа цепей: последовательные и параллельные. Элементы могут быть подключены последовательно, параллельно или их комбинация. В последовательной цепи электронов движутся только по одному пути. Здесь будет тот же ток, который проходит через каждый резистор.Напряжение на резисторах при последовательном соединении будет другим. Последовательные цепи нелегко перегреть. Конструкция последовательной схемы проста по сравнению с параллельной схемой.

В параллельной цепи электронов проходят через множество ее ветвей. В этом случае напряжение на каждом резисторе в цепи остается неизменным. Здесь ток в цепи делится между каждой ветвью и, наконец, рекомбинирует, когда ветви встречаются в общей точке. Параллельная цепь может быть сформирована разными способами, что означает, что ячейки могут быть расположены в различных формах.Параллельные цепи можно использовать в качестве делителя тока. Легко подключить или отключить новую ячейку или другой компонент, не затрагивая другие элементы в параллельной цепи. Но он использует много проводов и, следовательно, становится сложным.

Комбинация ячеек в последовательном соединении

Рассмотрим две последовательно соединенные ячейки. Положительный вывод одной ячейки подключается к отрицательной клемме следующей ячейки. Здесь один терминал двух ячеек свободен, а другой терминал двух ячеек соединен вместе.ε 1 и ε 2 — это ЭДС ячеек, а r 1 и r 2 — внутреннее сопротивление элементов соответственно. Пусть I будет током, протекающим через ячейки.

Ячейки, соединенные последовательно

Рассмотрим точки A, B и C, и пусть V (A), V (B) и V (C) — потенциалы этих точек соответственно. V (A) — V (B) будет разностью потенциалов между положительной и отрицательной клеммами для первой ячейки.

Так V AB = V (A) — V (B) = ε 1 — Ir 1.

V BC = V (B) — V (C) = ε 2 — Ir 2.

Теперь разность потенциалов между клеммами A и C равна

.

В AC = V (A) — V (C) = [V (A) — V (B)] + V (B) — V (C)]

= ε 1 — Ir 1 + ε 2 — Ir 2

= (ε 1 + ε 2 ) — I (r 1 + r 2 ).

В случае, если мы заменим эту комбинацию ячеек одной ячейкой между точками A и C с ЭДС ε экв и внутренним сопротивлением r экв, В AC = ε экв — r экв . и, таким образом, мы обнаружили, что ε eq = ε 1 + ε 2 и r eq = r 1 + r 2 из предыдущего уравнения.

Ясно, что эквивалентная ЭДС n ячеек в последовательной комбинации является суммой их индивидуальных ЭДС.Эквивалентное внутреннее сопротивление n ячеек в последовательной комбинации является суммой их индивидуального внутреннего сопротивления.

В последовательной комбинации, если ток покидает ячейку от отрицательного электрода, ЭДС ячейки будет, например, V BC = — ε 2 — Ir 2 и, наконец, уравнение для ε eq = ε 1 — ε 2 , (ε 1 > ε 2 ).

Преимущества и недостатки последовательно соединенных ячеек:

Ячейки, соединенные последовательно, создают большее результирующее напряжение.Поврежденные элементы можно легко идентифицировать, и, следовательно, их можно легко заменить, поскольку они разрывают цепь.

Повреждение одной из ячеек в цепи может повлиять на все соединение. Ячейки, которые соединены последовательно, быстро истощаются, и поэтому они не служат дольше. В домашней электропроводке не используется.


Комбинация ячеек при параллельном подключении

Рассмотрим две ячейки, соединенные параллельно. Здесь положительные выводы всех ячеек соединены вместе, а отрицательные выводы всех ячеек соединены вместе.При параллельном подключении ток делится между ответвлениями. Таким образом, ток I делится на I 1 и I 2. I = I 1 + I 2. Рассмотрим точки B 1 и B 2 , а затем V (B 1 ) и V (B 2 ) — потенциалы соответственно. Разность потенциалов на выводах первой ячейки.

Ячейки соединены параллельно

V = V (B 1 ) — V (B 2 ) = ε 1 — I 1 r 1. Точка B 1 и B 2 подключаются аналогично второй ячейке.

V = V (B 1 ) — V (B 2 ) = ε 2 — I 2 r 2 . По закону Ома мы знаем, что I = V / R. Теперь подставим эти значения в уравнение

.

Если мы заменим ячейки одной ячейкой, расположенной между точкой B 1 и B 2 с ЭДС ε экв и внутренним сопротивлением r экв , то V = ε экв — Ir экв .

Это то же самое, что и при параллельном соединении резисторов.

Для n количества ячеек, соединенных параллельно с ЭДС ε 1, ε 2 …… ε n и внутренним сопротивлением r 1 , r 2…. r n

Преимущества и недостатки параллельного подключения ячеек:

Для ячеек, соединенных параллельно, повреждение одной из ячеек в цепи не повлияет на все соединение.Ячейки, соединенные параллельно, не изнашиваются легко и, следовательно, служат дольше.

Напряжение, развиваемое ячейками при параллельном соединении, не может быть увеличено путем увеличения количества ячеек, присутствующих в цепи. Это потому, что они не имеют одинаковой круговой траектории. При параллельном подключении подключение обеспечивает питание из расчета на одну ячейку. Так что яркость лампочки не будет высокой.

Сводка
  • Алессандро Вольта изобрел электрическую батарею и впервые был назван гальванической батареей.

  • Фредерик Даниэлл разработал ячейку Даниэля, а Джордж Лекланш изобрел влажную ячейку. Доктор Карл Гасснер представил сухую батарею, а Гастон Планте представил первую перезаряжаемую батарею.

  • Элемент или батарея — это источник энергии, преобразующий химическую энергию в электрическую.

  • Батарея состоит из двух клемм. Положительный вывод называется Катод , а отрицательный вывод.называется Анод .

  • ЭДС или электродвижущая сила — это разность потенциалов, возникающая между двумя выводами батареи в разомкнутой цепи. ε = E / Q, где ε — электродвижущая сила, E — энергия, а Q — заряд. Внутреннее сопротивление — это сопротивление внутри батареи, которое препятствует прохождению тока при подключении к цепи. Уравнение, связывающее ЭДС и внутреннее сопротивление: V = ε — Ir, где V — разность потенциалов в цепи, ε — ЭДС, I — ток, протекающий по цепи, r — внутреннее сопротивление

  • Для двух последовательно соединенных ячеек развиваемое напряжение равно V = (ε 1 + ε 2 ) — I (r 1 + r 2 ).V = ε eq — r eq , если мы заменим количество ячеек одной ячейкой.

  • Для двух параллельно соединенных ячеек V = ε 1 r 2 + ε 2 r 1 / r 1 + r 2 — Ir 1 r 2 / r 1 + r 2. Для n количества ячеек, соединенных параллельно ε eq / r eq = ε 1 / r 1 + ε 2 / r 2 + ……………… …….. ε n / r n.


Посмотрите это видео, чтобы получить дополнительную информацию


Дополнительная информация

Ячейки, подключенные последовательно и параллельно

Возможно, вам понравится:

Neet coaching | Коучинг IIT в Дели | Лучший институт коучинга для ИИТ

Внутреннее сопротивление батарейного калькулятора • Электрические, радиочастотные и электронные калькуляторы • Онлайн-преобразователи единиц

Этот калькулятор определяет внутреннее сопротивление электрической батареи на основе падения напряжения на нагрузочном резисторе с известным сопротивлением и напряжения или тока холостого хода. в нагрузочном резисторе.

Пример 1: Рассчитайте внутреннее сопротивление Li-PO батареи, если ее напряжение без нагрузки составляет 3,90 В, а с нагрузкой 10 Ом — 3,89 В. Ниже вы найдете еще пять примеров.

Рассчитать

R I и I из U NL , R L и U L R

9013 L 909 909 U140 U NL , R L и I R I и R L от U NL , U 406

L L и I от U NL , R I и R L R L и I от UL I и U L R L и U L от U NL , R I и I N 906 37 I от R I , R L и U L U NL и U L от R I 906 и I U NL и R L от R I , U L и I

72 Напряжение на аккумуляторе, без нагрузки NL микровольт (мкВ) милливольт (мВ) вольт (В) киловольт (кВ) мегавольт (МВ)

Внутреннее сопротивление батареи

R I миллиом (мОм) килоом (мОм) кОм) мегом (МОм)

Для расчета введите любые три из пяти значений и нажмите или коснитесь кнопки Рассчитать .Исключение: при вводе только трех параметров нагрузки R L , U L и I невозможно рассчитать параметры батареи U NL и R I и никаких расчетов выполняются.

Определения и формулы

Согласно теореме Гельмгольца – Тевенина любую линейную сеть с любым количеством источников напряжения (например, шесть гальванических элементов, соединенных последовательно в автомобильном аккумуляторе), можно заменить электродвижущей силой (ЭДС ) или эквивалентное напряжение холостого хода U Источник NL последовательно с внутренним сопротивлением R I или полным сопротивлением Z I .Напряжение U NL питает внешнюю нагрузку R L током I .

Ток, подаваемый батареей на нагрузку, будет определяться сопротивлением внешней нагрузки, и в то же время этот ток будет ограничен внутренним сопротивлением батареи. Внутреннее сопротивление складывается из сопротивления пластин аккумулятора, его активного материала и электролита.

Свинцово-кислотные батареи имеют очень маленькое внутреннее сопротивление (обычно 0.01 Ом) — именно поэтому они способны обеспечить большой ток, необходимый для запуска двигателя. Внутреннее сопротивление свинцово-кислотных ячеек настолько мало, потому что в каждой ячейке есть несколько отрицательных и положительных пластин, соединенных параллельно. Кроме того, расстояние между отрицательной и положительной пластинами очень мало, и, следовательно, толщина слоя электролита между ними уменьшается, что, в свою очередь, делает их внутреннее сопротивление еще меньше. Когда батарея выдает большой ток, это внутреннее сопротивление рассеивает тепло, и батарея нагревается.

Внутреннее сопротивление батареи можно рассчитать по ее напряжению холостого хода U NL , напряжению, измеренному на нагрузке U L , и сопротивлению нагрузки R L . Это напряжение холостого хода эквивалентно электродвижущей силе батареи.

Ток, протекающий через нагрузочный резистор:

Падение напряжения на внутреннем сопротивлении:

Внутреннее сопротивление:

Полная формула:

В качестве альтернативы внутреннее сопротивление батареи можно рассчитать по току I L через сопротивление нагрузки, напряжение холостого хода аккумулятора и сопротивление нагрузки.

Напряжение на нагрузочном резисторе

Падение напряжения на внутреннем сопротивлении:

Внутреннее сопротивление:

Полная формула:

Как измерить внутреннее сопротивление батареи

As Как мы объяснили выше, для определения внутреннего сопротивления нам нужны три значения:

  • напряжение холостого хода батареи U NL , напряжение, измеренное на нагрузке U L , и сопротивление нагрузки R L

или

  • ток I L через сопротивление нагрузки, напряжение холостого хода батареи U NL и сопротивление нагрузки R L .

Чтобы правильно определить внутреннее сопротивление, нужно произвести несколько измерений с разными резисторами. Кроме того, внутреннее сопротивление может варьироваться в зависимости от температуры, возраста батареи и ряда других факторов. Таким образом, ваше измерение является лишь приблизительным, и не существует такого понятия, как «истинное» внутреннее сопротивление, которое можно было бы точно измерить.

На внутреннее сопротивление батарей влияют несколько факторов, включая их емкость, химический состав, качество элементов, возраст, температуру и скорость разряда.Дополнительную информацию о батареях вы найдете в наших калькуляторах энергии и времени работы батареи и калькуляторе батарей Drone LiPo.

Для измерения напряжения на нагрузке, подключенной к аккумулятору , вольтметр подключается параллельно нагрузке или клеммам аккумулятора. Если сопротивление нагрузки относительно низкое по сравнению с внутренним сопротивлением измерителя, вы получите достаточно точное показание напряжения нагрузки.

Для измерения тока , подаваемого на нагрузку, подключенную к батарее , между нагрузкой и батареей подключается амперметр, как показано на рисунке.Если его внутреннее сопротивление относительно мало по сравнению с сопротивлением нагрузки, вы можете предположить, что ваши измерения точны.

Конечно, теоретически и даже практически (например, для угольно-цинковой батареи) ток короткого замыкания батареи измерить путем короткого замыкания батареи амперметром вполне возможно. Однако, если батарея способна выдавать значительный ток, она может перегреться или даже загореться при коротком замыкании. Литий-ионные батареи могут даже взорваться, если их клеммы закорочены.Следовательно, ток почти всегда измеряется, когда батарея подключена к разумной нагрузке.

Для измерения напряжения холостого хода аккумуляторной батареи вольтметр подключается к ее клеммам без какой-либо нагрузки. Это напряжение также называется напряжением холостого хода. Если внутреннее сопротивление вольтметра намного выше внутреннего сопротивления аккумулятора, можно предположить, что напряжение холостого хода измеряется относительно точно.

Также необходимо измерить сопротивление нагрузки , если вы не используете прецизионный резистор.Помните, что если нагрузочный резистор нагревается, его сопротивление увеличивается, поэтому измерение тока необходимо проводить быстро.

Теперь вы можете поместить результаты ваших измерений в наш калькулятор и получить внутреннее сопротивление вашей батареи. Конечно, в продаже имеется множество специальных измерителей внутреннего сопротивления. Кроме того, более мощные зарядные устройства могут измерять внутреннее сопротивление батареи.

Чтобы получить полную картину, мы можем отметить, что каждая батарея имеет спектр внутренних сопротивлений или, скорее, импедансов, и для их измерения часто используется более сложная схема, которая питается от источника переменного тока с частотой изменяется от очень низкого до нескольких килогерц.Внутреннее сопротивление обычно характеризуется графиками, показывающими его зависимость от различных факторов.

Примеры расчетов

Пример 2. Батарея с ЭДС = 14,5 В выдает 25 Вт мощности на внешний нагрузочный резистор. Напряжение на выводе аккумулятора составляет 11,9 В. Определите внутреннее сопротивление аккумулятора. Намек. Воспользуйтесь нашим калькулятором закона Ома, чтобы определить ток через нагрузочный резистор. Затем используйте этот калькулятор для определения внутреннего сопротивления.

Пример 3. Лампа накаливания на 4 Ом подключена к батарее с внутренним сопротивлением 0,15 Ом. Вольтметр, подключенный к клеммам аккумулятора, показывает 11,5 В. Какая ЭДС аккумулятора?

Пример 4. Две галогенные лампы головного света мощностью 55 Вт подключены параллельно к клеммам аккумуляторной батареи грузового автомобиля, имеющей внутреннее сопротивление 0,02 Ом. Напряжение на выводах аккумулятора составляет 23,6 В. Какая ЭДС аккумулятора? Подсказка: используйте наш калькулятор мощности постоянного тока, чтобы определить сопротивление горячей лампы.Затем используйте наш калькулятор параллельного сопротивления, чтобы определить сопротивление двух параллельно соединенных ламп. Затем используйте этот калькулятор для определения ЭДС аккумулятора.

Пример 5. Определите ток короткого замыкания автомобильного аккумулятора на 12 В с ЭДС = 13,5 В и внутренним сопротивлением 0,04 Ом. Подсказка: 12 В — это номинальное напряжение аккумулятора, и это число не используется при решении этой проблемы.

Пример 6. Аккумулятор с ЭДС = 1.5 В замкнут накоротко с помощью неидеального амперметра с внутренним сопротивлением 0,02 Ом, которое показывает 2,7 А. Определите его внутреннее сопротивление и мощность, рассеиваемую внутри батареи. Подсказка: сначала используйте этот калькулятор для определения внутреннего сопротивления батареи, а затем воспользуйтесь нашим калькулятором мощности постоянного тока, чтобы определить мощность, рассеиваемую в батарее.

Пример 7. Контроллер запуска модели, который используется для запуска ракетного двигателя путем нагрева нихромовой проволоки воспламенителя, питается от четырех АА 1.Батареи 5 В, подключенные последовательно. Каждая батарея имеет внутреннее сопротивление 200 мОм. Сопротивление двух разных воспламенителей ракетных двигателей составляет 0,7 и 3 Ом. Определите ток, подаваемый на воспламенитель 0,7 Ом и воспламенитель 3 Ом. Подсказка: напряжение четырех последовательно соединенных батарей составляет 1,5 × 4 = 6 В, а их общее внутреннее сопротивление составляет 200 × 4 = 0,8 Ом.

Эту статью написал Анатолий Золотков

Лекция 15

Лекция 15 Сводка
  • Электрический ток
  • Электропроводность и удельное сопротивление
  • Закон Ома
    Сегодняшняя шутка
  • Written Quiz Ch.25
  • Электроэнергия


  • Стоимость электроэнергии

  • Пример # 4
  • ЭДС и внутреннее сопротивление

  • Водяная модель для контуров

ПОП4 21,15
Если генератор мощностью 1500 л.с. на 80% эффективен при преобразовании механической энергии в электрическую, что ток он выдает при напряжении на клеммах 2000 В?
А. 0,600 А
Б.125 А
С. 280 А
D. 448 A
Ответ

PSE6 27,50
Сколько времени нужно кофеварке на 120 В, 2,00 А, чтобы нагреть 0,500 кг воды с 23 ° C до 100 ° C?
A. 67,2 с
Б. 7,55 мин.
C. 11,2 мин.
Д. 38 мин.
Ответ

POP5 21.44c
75 Вт и коммуникатор; Лампа на 120 В подключается к удлинителю двумя проводами по 0,800 Ом каждый. Какая мощность фактически передается лампе?
А.75.0 Вт
Б. 74,5 Вт
С. 73,8 Вт
D. 72.2 W
Ответ

PSE6 27,51
Примерно сколько стоит использовать 700-ваттную кофеварку каждое утро в течение года? Предположим, что электроэнергия стоит около 0,10 доллара США / кВтч.
1
австралийских долларов Б. $ 10
100 долл. США
D. $ 1000
Ответ

POP5 21,33
Батарея имеет ЭДС 15,0 В. Напряжение на клеммах батареи составляет 11,6 В при подаче напряжения. 20.0 Вт мощности на внешний нагрузочный резистор. Какое внутреннее сопротивление батареи?
А. 1,97 Ом
Б. 3,18 Ом
C. 5,76 Ом
D. 6,73 Ом
Ответ

Д. 448 А

C. 11,2 мин.

C. 73,8 Вт


Б. $ 10



Предполагается, что вы включаете кофеварку на 20 минут каждое утро, а цена 0 $.10 / кВтч.

А. 1,97 Ом


электричество — Расчет энергии, отдаваемой батареей

электричество — Расчет энергии, отдаваемой батареей — Physics Stack Exchange
Сеть обмена стеков

Сеть Stack Exchange состоит из 178 сообществ вопросов и ответов, включая Stack Overflow, крупнейшее и пользующееся наибольшим доверием онлайн-сообщество, где разработчики могут учиться, делиться своими знаниями и строить свою карьеру.

Посетить Stack Exchange
  1. 0
  2. +0
  3. Авторизоваться Подписаться

Physics Stack Exchange — это сайт вопросов и ответов для активных исследователей, ученых и студентов-физиков.Регистрация займет всего минуту.

Зарегистрируйтесь, чтобы присоединиться к этому сообществу

Кто угодно может задать вопрос

Кто угодно может ответить

Лучшие ответы голосуются и поднимаются наверх

Спросил

Просмотрено 52к раз

$ \ begingroup $

Если вы хотите рассчитать энергию, выделяемую батареей за время $ t $, вы должны использовать $ E = VIt $, где $ I $ — ток через батарею.2} {r} = VI $ или $ V = Ir $. Но в целом это неверно (в качестве примера возьмем ЭДС $ 12В $, ток $ 0,5A $ и сопротивление нагрузки $ 15 \ Omega $), и мне интересно, почему это не работает.

Спасибо!

задан 14 мая ’15 в 17: 122015-05-14 17:12

пользователь 45220

1,9166 золотых знаков2626 серебряных знаков4343 бронзовых знака

$ \ endgroup $ $ \ begingroup $

Ваш анализ не применим к тому, к чему вы его применяете.2 / R $, где необходимо убедиться, что $ V $ в этом уравнении является фактическим напряжением на резисторе. Возможно, вам придется принять во внимание, что часть ЭДС батареи будет проявляться на внутреннем сопротивлении, поэтому напряжение на внешнем сопротивлении будет несколько меньше, чем ЭДС батареи. Подробности оставлены читателю.

alexxlab

Добавить комментарий

Ваш адрес email не будет опубликован. Обязательные поля помечены *